Está en la página 1de 34

Neurology Questions & Answers

Neurology:Question 1
A 39-year-old executive comes to the office for a follow-up visit. She had a generalized tonic-clonic seizure 1
week ago, witnessed by her husband, who reports that the seizure started suddenly. She had tonic extension
and stiffening of her whole body followed by rhythmic jerking movements of her arms, trunk, and legs. The
patient had no warning prior to the seizure, and afterward fell into a deep sleep. She was hospitalized. Wake
and sleep electroencephalography and magnetic resonance imaging of the brain with contrast were normal, and
she was sent home on a maintenance dose of phenytoin. The patient states that, for the 4 nights prior to the
seizure, she slept no more than 2 hours a night in order to finish an assignment. There is no personal or family
history of seizures. Neurologic examination is normal.
Which of the following is the most appropriate next step in management?
A. Continue phenytoin
B. Discontinue phenytoin
C. Discontinue phenytoin and begin carbamazepine
D. Discontinue phenytoin and begin valproic acid
E. Discontinue phenytoin and begin gabapentin
Neurology:Question 2
A 73-year-old woman has transient left hemiparesis lasting 20 minutes and calls your office that same day to
inform you. She has a history of intermittent nonvalvular atrial fibrillation, angina pectoris, and hypertension.
At the time of her call, she is completely asymptomatic. Her only antithrombotic therapy is aspirin, 81 mg daily.
What is the optimal initial management for this patient?
A. Schedule an office appointment within 1 week.
B. Schedule transesophageal echocardiography.
C. Schedule Holter monitoring.
D. Schedule outpatient carotid duplex ultrasonography.
E. Obtain emergency computed tomography.
Neurology:Question 3
A 35-year-old woman comes to the office with a 1-day history of blurred vision and retro-orbital pain with
movement of the right eye. Past medical history is unremarkable. On examination, she has a right afferent
pupillary defect. Visual acuity is 20/200 in the right eye with a dense central scotoma, and 20/25 in the left eye.
Funduscopic examination is normal bilaterally. The rest of the neurologic examination is normal. Magnetic
resonance imaging of the brain shows one nonenhancing periventricular white matter lesion. The patient
refuses lumbar puncture for examination of cerebrospinal fluid. Treatment with intravenous
methylprednisolone improves her optic neuritis.
Which of the following should be done to monitor for development of multiple sclerosis?
A. Repeat examination only when patient has a new neurologic symptom
B. Serial visual evoked potential studies in 6 months
C. Formal visual field studies in 6 months
D. Follow-up magnetic resonance imaging of the brain with gadolinium in 6 months
E. Serial nerve conduction velocity examinations in 6 months
Neurology:Question 5
A 36-year-old woman with a 15-year history of headaches comes to the office for a follow-up visit. Initially, she
had moderate to severe bilateral pulsatile, frontal headaches accompanied by nausea, light and noise
sensitivity, and occasional vomiting. They occurred one to two times each month and lasted 24 to 36 hours. In
the last 2 years, she has developed a daily mild to moderate bilateral frontal headache that is present when she
awakens and lasts all day. Approximately once per week she has a headache that is similar to her previous
episodic headaches but less severe. She takes amitriptyline, 75 mg at bedtime, as a prophylactic agent. She takes
acetaminophen/caffeine/butalbital, 4 tablets/d, and rizatriptan, 10 to 20 mg, one to two days per week for
more severe headaches. Physical and neurologic examinations are normal.
What is the most appropriate next step in managing this patients headache disorder?
A. Discontinue amitriptyline and initiate treatment with divalproex sodium.
B. Discontinue acetaminophen/caffeine/butalbital.
C. Discontinue rizatriptan.
D. Increase dose of amitriptyline.
Neurology:Question 6
A 65-year-old man has weakness and wasting of his left hand and muscle twitching involving his arms
and thighs. The symptoms have progressed over 5 months. There is no associated pain or paresthesia.

Neurologic examination reveals severe weakness and atrophy of the left thenar and hypothenar muscles, and
mild weakness of left foot dorsiflexion. Muscle stretch reflexes are hyperactive and symmetric. Sensation is
normal. Magnetic resonance imaging of the cervical spine was obtained by an orthopedic surgeon 2 weeks
earlier and is normal.
Which of the following is the most appropriate next step?
A. Muscle biopsy
B. Nerve biopsy
C. Serum creatine kinase determination
D. Electromyography
E. Genetic testing
Neurology:Question 7
A 46-year-old woman comes to the office for management of a 4-mm left middle cerebral artery aneurysm. The
aneurysm was found when the patient had magnetic resonance angiography and imaging for evaluation of a
chronic headache disorder. Medical history is limited to chronic migraine headaches and smoking. Neurologic
examination is normal.
Which of the following is the most appropriate next step in this patients management?
A. Arteriography
B. Computed tomographic angiography
C. Follow-up magnetic resonance angiography in 6 months
D. Aneurysm clipping
Neurology:Question 8
A 71-year-old man comes to the office because of slowness, tremor, stiffness, and difficulty with dexterity, such
as buttoning clothes. The problems developed insidiously over the last few years, and have caused him to curtail
his activities severely. On examination, he has a shuffling gait, tremor of one hand while walking, and reduced
arm swing (right more than left). Rapid alternating movements are very slow, especially in the right limbs.
Passive movement of his limbs reveals rigidity. Neurologic examination is otherwise normal.
Which of the following is the most appropriate management for this patient?
A. No treatment
B. Selegiline
C. Amantadine
D. Carbidopa/levodopa
E. Entacapone
Neurology:Question 9
A 68-year-old man is under evaluation for memory difficulty that, according to his wife, began insidiously 3 or
4 years earlier. He has difficulty remembering recent events. For example, he forgets appointments and recent
conversations, and forgot that a close relative had recently died. He is no longer able to manage his own
checkbook or operate his car without getting lost. Medical history is unremarkable. Physical examination is
normal. Mental status examination shows prominent memory loss and difficulty drawing a complex figure.
Magnetic resonance imaging of the brain shows only mild cerebral atrophy.
Which of the following is the most likely diagnosis?
A. Alzheimers disease
B. Dementia with Lewy bodies
C. Vascular dementia
D. Frontotemporal dementia
E. Creutzfeldt-Jakob disease
Neurology:Question 10
A 23-year-old woman comes to the office with her husband for evaluation of recent onset of unusual behavior.
While providing the clinical history, the patient begins to cry. Shortly after that the patients arms start shaking.
Gradually, the shaking becomes more pronounced and involves her legs and head. The patients breathing
becomes deep and fast. This continues on and off for several minutes. The shaking stops gradually. The patient
opens her eyes and states that she has a headache and is very tired and dizzy. Family history is positive for
seizures in the patients cousin. There has been a significant amount of family stress recently.
Which of the following is the most likely diagnosis?
A. Syncope
B. Nonepileptic event (pseudoseizure)
C. Partial seizure with secondary generalization
D. Generalized tonic-clonic seizure
E. Basilar migraine
Neurology:Question 11
A 65-year-old woman comes to the emergency department because of mild left hem iparesis and mild left-sided

sensory deficit that began suddenly 2 days earlier. She has a history of myocardial infarction and takes aspirin,
81 mg daily.
Which of the following is the most appropriate next step in the management of this patient?
A. Increase the aspirin dose to 325 mg daily.
B. Discontinue aspirin and begin clopidogrel, 75 mg daily.
C. Change aspirin to combined low-dose aspirin/extended-release dipyridamole.
D. Discontinue aspirin and begin warfarin (target international normalized ratio, 2.0 to 3.0).
E. Initiate inpatient diagnostic evaluation.
Neurology:Question 12
A 74-year-old man with a history of hypertension is brought to the emergency department after being found
unresponsive by his wife in bed. He is intubated for airway protection. Blood pressure is 210/90 mm Hg; pulse,
80/min; and temperature, 37 C (98.6 F). When his eyes are opened by the examiner, there are intermittent
spontaneous downward eye movements. There are no spontaneous horizontal eye movements, nor are there
horizontal movements in response to the dolls head maneuver. The pupils are very small but reactive. He does
not move any extremities on command. There is occasional spontaneous extensor posturing in his arms and
legs, and Babinskis signs are present bilaterally. He blinks his eyes three times in a row on command.
Which of the following is the most likely cause of this patients condition?
A. Anoxic encephalopathy
B. Left middle cerebral artery stroke
C. Pontine infarct
D. Cerebellar infarct
Neurology:Question 13
A 42-year-old man with secondary progressive multiple sclerosis comes to the local urgent care facility because
of increased weakness and spasticity of the legs, with increased hip flexor tone. Over the past 3 days he has also
noted an increase in urinary urgency and frequency. Until 3 days ago, he had been fairly stable, with a slowly
progressive course, and could walk at least 500 feet with one cane; spasticity had been well controlled with
baclofen, 20 mg three times a day orally. On examination, blood pressure is 110/70mm Hg; pulse, 92/min; and
temperature, 38.2 C (100.8 F). He has bilateral internuclear ophthalmoplegia. Hip flexion is weak bilaterally.
Spasticity is severe in the legs, with sustained clonus at both ankles. Urinalysis shows 50 to 100 leukocytes/hpf,
and 2+ leukocyte esterase.
In addition to treatment of the urinary tract infection, which of the following is the most appropriate
treatment?
A. Acetaminophen
B. Oral prednisone
C. Intravenous methyiprednisolone
D. Interferon beta
E. Glatiramer acetate
Neurology:Question 14
A 34-year-old woman comes to the office because of a 2-year history of bifrontal, throbbing headaches that
occur two to four times per month and last 8 to 16 hours. During a headache, she tolerates noise and sound
poorly and prefers to lie down. She sometimes has associated blurred vision, eye tearing, and rhinorrhea, and
she is occasionally unable to work during a headache. The headache worsens with physical activity, such as
brisk walking. Physical and neurologic examinations, including funduscopic examination, are normal.
Which of the following is the most likely diagnosis?
A. Episodic tension-type headache
B. Cluster headache
C. Idiopathic intracranial hypertension
D. Migraine without aura
E. Sinus headache
Neurology:Question 15
A 55-year-old woman presents to her physician with a 4-week history of occipital headache with morning
vomiting and a 1-week history of difficulty with peripheral vision to the left. She reports no other neurologic
symptoms. The patient has a history of node-positive breast cancer. Neurologic examination reveals
papilledema, left homonymous hemianopia, mild left hemiparesis, and subtle gait ataxia. Chest radiograph
shows multiple nodules.
Which of the following is the most likely cause of this patients findings?
A. Parenchymal brain metastases
B. Leptomeningeal metastases
C. Paraneoplastic encephalitis
D. Cerebral infarction

Neurology:Question 16
A 25-year-old woman comes to the office for a follow-up visit. She has a 10-year history of complex partial
seizures with occasional secondary generalization. The patient reports that she is 14 weeks pregnant and that
she has had no seizures since her last visit more than 1 year ago. Currently, she is on carbamazepine
monotherapy, 200 mg three times daily. Her carbamazepine blood level today is 8.2 glmL (therapeutic range,
8 to 12 glmL).
Which of the following is the most appropriate next step in management?
A. Continue carbamazepine at the current dose
B. Increase the carbamazepine dose
C. Add another antiepileptic agent
D. Change carbamazepine to lamotrigine
E. Discontinue carbamazepine
Neurology:Question 17
A 67-year-old man comes to the emergency department because of sudden onset of severe vertigo, headache,
nausea, vomiting, and inability to walk. He has a history of hypertension. On examination, his blood pressure is
190/110 mm Hg; pulse, 100/min, respiratory rate, 18/mm, and temperature, 36.7 C (98 F). He is in distress
from severe headache and nausea. He has dysarthria, prominent nystagmus on lateral gaze, and clumsiness of
the left arm on finger-to-nose maneuver, and he falls to the left when he tries to sit up.
Which of the following is the most likely cause of this patients presentation?
A. Menieres disease
B. Vestibular neuronitis
C. Cerebellar hemorrhage
D. Benign positional vertigo
Neurology:Question 18
A 38-year-old man comes to the office because of paresthesias in his feet that began approximately 8 months
ago. He now also has bilateral foot drop and weakness of the right hand. On examination, he has severe
weakness in the foot dorsiflexors bilaterally and moderate weakness of foot eversion, inversion, and plantar
flexion. He has mild weakness of hip flexion and is unable to rise from a chair without using his arms. There is
also weakness in the ulnar-innervated hand muscles, slightly more pronounced on the right than on the left.
There are no obtainable reflexes, and he has moderate proprioceptive loss in the toes.
Which of the following is the most likely diagnosis?
A. Guillain-Barr syndrome
B. Chronic inflammatory demyelinating polyneuropathy
C. Polymyositis
D. Myasthenia gravis
Neurology:Question 19
An 85-year-old man arrives in the emergency department within 30 minutes of sudden-onset left
hemiparesis. Blood pressure is 240/124 mm Hg on arrival and 2 hours after the onset of his symptoms.
He has severe left hemiparesis, neglect (left hemi-inattention), and a left hemisensory deficit.
Electrocardiography, laboratory tests, and cerebral computed tomography are normal.
Which of the following is the most appropriate management for this patient?
A. Give intravenous labetalol to achieve blood pressure below 185/110 mm Hg and then give intravenous tissue
plasminogen activator (t-PA).
B. Give intravenous nitroprusside to achieve blood pressure below 185/110 mm Hg and then give intravenous tPA.
C. Do not give antihypertensive medication or intravenous t-PA.
D. Do not give antihypertensive medication but do give intravenous t-PA.
E. Give antihypertensive medication to achieve blood pressure below 185/110 mm Hg and then give intraarterial t-PA.
Neurology:Question 20
A 23-year-old woman comes to the office for re-evaluation of migraine headaches. She was seen 2 weeks earlier
because of recurrent generalized, moderate to severe pulsatile headaches associated with nausea and light and
noise sensitivity. The headaches typically last 1 to 2 days, and she was having approximately 10 headache days
per month. Amitriptyline was begun at a dose of 10 mg at bedtime, with instructions to increase the dose in 10mg increments every week toward an initial target dose of 50 mg. The current dose is 30 mg at bedtime. She
tolerates the amitriptyline well. Today she has developed a typical attack that is treated with subcutaneous
sumatriptan in the office.
What plan should be pursued for prophylactic treatment?
A. Continue amitriptyline at the current dose.
B. Continue increasing amitriptyline as planned.

C. Discontinue all prophylaxis.


D. Discontinue amitriptyline and begin propranolol.
E. Discontinue amitriptyline and begin divalproex sodium.
Neurology:Question 21
A 55-year-old man reports that his handwriting has been getting smaller, and his wife states that he has been
walking with a slow, shuffling gait for about 2 years. He otherwise feels well. He has diabetes, diabetic
gastroparesis, a 3-year history of celiac disease responsive to medications and a gluten-free diet, and
depression. Daily medications are glyburide, ranitidine, metoclopramide, and bupropion. On examination, he
has facial masking, a slightly shuffling gait, bilateral reduced arm swing, mild diffuse limb rigidity, and reduced
speed when performing rapid alternating movements.
What is the most appropriate management of his current symptoms?
A. Discontinue metoclopramide.
B. Discontinue ranitidine.
C. Discontinue bupropion.
D. Begin carbidopa/levodopa.
E. Begin pramipexole.
Neurology:Question 22
A 60-year-old woman with a history of adenocarcinoma of the lung presents to her physician with a 3-week
history of horizontal diplopia, dysphagia, and right leg pain with foot drop. Examination reveals weakness of
right eye abduction, left facial weakness, reduced gag reflex, and weakness of the right leg muscles supplied by
the L5 nerve root. Computed tomography of the head with contrast shows only enlargement of the entire
ventricular system (communicating hydrocephalus).
Which of the following is the most likely cause of this patients findings?
A. Parenchymal brain metastases
B. Venous sinus thrombosis
C. Paraneoplastic encephalitis
D. Leptomeningeal metastases
E. Herpes zoster infection of the central nervous system
Neurology:Question 23
A 21-year-old woman comes to the office with her mother because of recurrent blackouts. The patient has had
five episodes in her life, the last one a week ago. She describes feeling dizzy, lightheaded, nauseated, and warm
prior to blacking out. According to her mother, the patient becomes pale, then falls suddenly to the ground and
has several jerks of her limbs. Her mother states that the patient recovers consciousness almost immediately,
without any confusion or disorientation. During one event, the patient was incontinent of urine, and did not
recover for at least 30 seconds. Most episodes have occurred in church after prolonged standing; the latest
occurred when the patient witnessed a medical procedure while visiting a relative in the hospital.
Which of the following is the most likely diagnosis?
A. Juvenile myoclonic epilepsy
B. Complex partial seizures with secondary generalization
C. Syncope
D. Nonepileptic events (pseudoseizures)
E. Atonic seizures
Neurology:Question 24
A 29-year-old woman with multiple sclerosis comes to the office because of overwhelming fatigue. She has had
three exacerbations of multiple sclerosis in the past 8 years and is now in remission. Her fatigue is particularly
severe in mid afternoon; she often naps if possible or finds it difficult to concentrate on work. She has no
problem falling asleep at night and does not wake up before she wishes. She sleeps 10 hours a day. She has no
symptoms to suggest depression. Neurologic examination and thyroid function studies are normal.
Which of the following would be a reasonable medication to prescribe for her pathologic fatigue?
A. Thyroid supplement
B. Amphetamine
C. Amantadine
D. Fluoxetine
E. Pemoline
Neurology:Question 25
A 78-year-old man is evaluated in the office for progressive impairment of memory and other aspects of
intellect, which has developed insidiously over 3 years. The patient recalls none of three words after a 3-minute
delay, and has a Mini-Mental State Examination score of 22 of 30. There are no other
significant findings on physical examination.
Which of the following is most likely to improve this patients symptoms?

A. Galantamine
B. Gingko biloba
C. Vitamin E
D. Selegiline
Neurology:Question 26
A 67-year-old man who lives alone is found on the floor of his home by his son and is brought to the emergency
department within 30 minutes. When last seen by his son 3 days earlier, the patient was in full health. In the
emergency department, his blood pressure is 180/100 mm Hg. He is mildly lethargic and has global aphasia, a
dense right visual field deficit, right hemiplegia, and no response to painful stimuli on the right side. Computed
tomography of the head shows that much of the left hemisphere is dark (hypodense) with mass effect
(compression of sulci and the left lateral ventricle).
Which of the following is the most appropriate next step in management of this patient?
A. Do not give acute therapy.
B. Give intravenous tissue plasminogen activator.
C. Give intra-arterial tissue plasminogen activator.
D. Give intravenous corticosteroids.
E. Give intravenous heparin.
Neurology:Question 27
A 60-year-old woman comes to the office for evaluation of fainting episodes. She feels lightheaded when she
stands, and she has fainted three times within the past 2 months. She has imbalance and a shuffling gait that
developed slowly over the past 2 years. She is sleepy during the day, and her husband describes high-pitched
snoring (screeching) while she sleeps. She has urinary incontinence. Her medical history is otherwise
unremarkable. She takes no medications. Her older sister has a gait disorder. On examination, blood pressure
is 120/80 mm Hg while sitting and 80/50 mm Hg when standing; pulse is 80 beats/min in both positions. Eye
movements are normal. Finger-to-nose and heel-to-shin movements are uncoordinated, but there is no tremor.
Movements are slow and there is limb rigidity. Deep tendon reflexes are brisk, and Babinskis sign is present.
Which of the following is the most likely diagnosis?
A. Parkinsons disease
B. Creutzfeldt-Jakob disease
C. Spinocerebellar degeneration
D. Progressive supranuclear palsy
E. Multiple system atrophy
Neurology:Question 28
A 46-year-old man comes to the office because of a 1-year history of a slowly progressive gait disorder. He
describes his legs as heavy, and he tires quickly when walking. He has spasms in his legs especially when they
are tired or when he tries to sleep at night, On examination, he has mild weakness in both hip flexors and ankle
dorsiflexors, diffuse hyperreflexia, and bilateral extensor plantar responses. He has severe loss of
proprioception and vibration sense in the legs. Pinprick sensation is normal throughout. Magnetic resonance
imaging of the cervical and thoracic spine is normal; cerebrospinal fluid is normal, and there are no oligoclonal
bands.
Which of the following is the most likely diagnosis?
A. Diabetic neuropathy
B. Polymyositis
C. Vitamin B 12 deficiency
D. Chronic inflammatory demyelinating polyradiculopathy
E. Lambert-Eaton myasthenic syndrome
Neurology:Question 29
A 29-year-old computer programmer is brought to the emergency department by friends after he suddenly falls
to the ground at a party. He has severe headache, photophobia, and vomiting. He has no significant medical
history. On examination, blood pressure is 210/110 mm Hg; pulse rate, 110/min; respiratory rate, 22/min; and
temperature, 37.2 C (99 F). He is in moderate distress from headache. He has moderate meningismus but no
focal neurologic findings. Routine laboratory tests are normal. Urine drug screen is positive for cocaine.
Computed tomography of the brain shows a moderate-sized left thalamic hemorrhage with some
intraventricular extension of blood. The patient is stabilized and his blood pressure is controlled with
intravenous nicardipine. He is admitted to the intensive care unit for monitoring.
What additional diagnostic testing is most appropriate?
A. None
B. Electroencephalography
C. Echocardiography
D. Cerebral angiography

Neurology:Question 30
An obese 66-year-old man has had increasing pain and numbness of the right anterior thigh over the past 3
weeks. He has not seen a physician in more than 20 years. On examination, he has moderate weakness of right
knee extension, thigh adduction, and hip flexion. The right knee jerk reflex is absent; the left knee jerk is 1+.
Achilles reflexes are absent bilaterally. He has sensory loss in the right anterior thigh region and a stockingglove distribution of sensory loss in the distal legs. Magnetic resonance imaging of the lumbosacral spine and
lumbosacral plexus is normal.
Which of the following studies is most likely to be diagnostic?
A. Creatine kinase level
B. Hemoglobin A 10 level
C. Thyroid function
D. Lumbar puncture
E. Erythrocyte sedimentation rate
Neurology:Question 31
A 68-year-old woman comes to the office because of a 2-year history of falls and imbalance that has progressed
insidiously. She also has trouble reading and doing needlework because of poor vision. Family history is
negative for neurologic disease. She has a staring facial expression. Visual acuity and visual fields are normal,
but she has impaired upward/downward gaze. She moves stiffly and slowly, and nearly falls over while walking.
Her neck and, to a lesser degree, limbs are rigid. Rapid alternating movements are markedly slowed.
Which of the following is the most likely diagnosis?
A. Parkinsons disease
B. Generalized dystonia
C. Multiple system atrophy
D. Progressive supranuclear palsy
E. Cerebellar degeneration
Neurology:Question 32
A 57-year-old man with a clerical job is brought to the office by his wife for evaluation of changes in memory,
personality, and behavior. Over the past 3 years, he has become progressively insensitive to his wifes concerns,
to the point that she has considered separation. He makes loud, inappropriate comments in public, and recently
received a poor review from his work supervisor. Past medical history is unremarkable; he takes no
medications. His mother, a maternal uncle, and the maternal grandfather had dementia. Physical examination
is normal. The Mini-Mental State Examination score is 28 of 30.
Which of the following is the most likely diagnosis?
A. Mild cognitive impairment
B. Alzheimers disease
C. Dementia with Lewy bodies
D. Frontotemporal dementia
E. Creutzfeldt-Jakob disease
Neurology:Question 33
A 70-year-old man presents to his physician with a 2-month history of progressive cognitive abnormalities and
right-sided weakness. He has difficulty dressing and bathing himself and is frequently disoriented with respect
to time and place. For the last week he is dragging his right foot when walking. On examination, he is alert but
inattentive. He is disoriented with respect to time and place. Papilledema is present. There is a mild right
hemiparesis affecting the right face, arm, and leg. Computed tomography of the head with contrast shows
multiple, homogeneously enhancing lesions in the periventricular white matter of both cerebral hemispheres
(left more than right) with surrounding edema and mass effect. A biopsy of one of the lesions is performed.
Postoperatively, dexamethasone, 4 mg three times daily, is administered; after 2 days, the patient is alert and
fully oriented, and his right-sided strength is normal. Computed tomography of the head is repeated and shows
marked improvement in all lesions with reduced enhancement and mass effect.
Which of the following is the most likely cause of this patients findings?
A. Metastatic carcinoma
B. Multiple sclerosis
C. Glioblastoma multiforme
D. Central nervous system vasculitis
E. Primary central nervous system lymphoma
Neurology:Question 34
A 40-year-old woman with secondary progressive multiple sclerosis continues to have a gradual decline in
neurologic status despite treatment with interferon beta. Two years ago she could walk 200 meters without a
cane or rest. She has recently worsened to the point that she can walk no further than 25 meters with a walker.
On examination, she has bilateral lower-extremity weakness and hyperreflexia. Magnetic resonance imaging of

the brain shows multiple white matter lesions in a periventricular distribution.


Which of the following treatments would be most appropriate instead of interferon beta?
A. Chronic oral prednisone
B. Bone marrow transplantation
C. Glatiramer acetate
D. Plasmapheresis
E. Mitoxantrone
Neurology:Question 35
A 24-year-old woman comes to the office because of severe generalized, throbbing headaches associated with
nausea, occasional vomiting, and light and noise sensitivity. The headaches occur two to three times weekly and
last 12 to 24 hours. She misses approximately 1 workday each month. She has been taking zolmitriptan, 5 mg
orally, for acute attacks, with good benefit approximately two thirds of the time. She has insomnia and a past
history of childhood asthma.
Which of the following medications is most appropriate to prevent the headaches?
A. Amitriptyline
B. Nadolol
C. Naratriptan
D. Propranolol
E. Sertraline
Neurology:Question 36
A 38-year-old man comes for a follow-up visit. Four years earlier, he had three complex partial seizures and
began treatment with phenytoin. He has had no seizures since beginning treatment and tolerates the
medication well. He has no previous personal or family history of seizures or other neurologic illness.
Neurologic examination is normal. The patients electroencephalograms (EEGs) consistently show right
temporal spikes and sharp waves; his last EEG was 3 months ago. Magnetic resonance imaging (MRI) of the
brain is normal. He asks whether he still needs to take phenytoin.
Which of the following would you recommend for this patient?
A. Taper phenytoin
B. Continue phenytoin at current dose
C. Change phenytoin to carbamazepine
D. Obtain single photon emission computed tomography (SPECT)
E. Repeat EEG and MRI of brain
Neurology:Question 37
A 50-year-old woman presents to her physician with a 4-month history of progressive right-sided headache that
sometimes awakens her from sleep. She has noted a non-tender swelling over the right side of her head for the
past year. This is mostly apparent to her when she combs her hair. Neurologic examination is normal except for
hyperreflexia of her left upper arm. There is asymmetry of the head, with a fullness of the right side of the skull.
Computed tomography of the head reveals a dural-based, calcified, enhancing mass measuring 6 cm along the
right cerebral convexity. There is hyperostosis of the skull contiguous with the lesion.
Which of the following is most likely responsible for the clinical and radiographic findings?
A. Low-grade glioma
B. Glioblastoma multiforme
C. Meningioma
D. Subdural hematoma
E. Dural metastasis
Neurology:Question 38
An 87-year-old previously healthy woman awakens at 5:00 a.m., makes coffee, and then falls to the kitchen
floor unable to speak or move her right side, as witnessed by her daughter. In the emergency department, her
blood pressure is 174/70 mm Hg. She has a dense global aphasia, right visual field deficit, right hemiplegia, and
right hemisensory loss. Electrocardiography, routine laboratory tests, and computed tomography of the head
are normal. At 6:15 a.m., the patients daughter asks about whether the patient should receive intravenous tissue
plasminogen activator (t-PA).
Which of the following is the most appropriate response?
A. Intravenous t-PA is contraindicated because of the patients age and size of stroke.
B. Intravenous t-PA offers the patient the best chance of recovery.
C. Intra-arterial t-PA is safer than intravenous t-PA.
D. Intravenous t-PA and oral aspirin given simultaneously offer the best chance for recovery.
Neurology:Question 39
A 33-year-old man is brought to the emergency department by ambulance after having three generalized tonicclonic seizures over 30 minutes without recovering consciousness between seizures. His friend states that the

patient has a history of epilepsy but is not compliant with his medications. During examination, the patient has
another generalized seizure. He becomes cyanotic despite oxygen supplementation. His pulse is 110/min and
blood pressure is 150/90 mm Hg. He continues to have seizures despite receiving 8 mg of intravenous
lorazepam en route to the hospital. The patient weighs approximately 80 kg (176 Ib).
Which of the following intravenous medications should be administered next?
A. Fosphenytoin 1600 mg phenytoin-equivalent
B. Phenobarbital 1600 mg
C. Phenytoin 1000 mg
D. Valproic acid 1600 mg
Neurology:Question 40
A 75-year-old man is brought to the office because he is confused on a regular basis, particularly at night, and
sometimes moment to moment during the day. He woke up thinking he was tied to the bed. He hallucinates
that people are in the house, and sometimes believes his wife has been replaced by a look-alike. On some days
he needs help with basic activities. Medical history is significant for a confusional episode due to prescription of
meclizine for acute peripheral vestibulopathy 2 years earlier. Meclizine was discontinued, and at a follow-up
office visit 1 month later the confusion was much improved. He currently takes no medications. There is no
family history of dementia or parkinsonism. Masked facies, mild bradykinesia, and rigidity are evident. His
score on the Mini-Mental State Examination is 20 of 30. The remainder of the examination is normal.
Which of the following is the most likely diagnosis?
A. Mild cognitive impairment
B. Alzheimers disease
C. Frontotemporal dementia
D. Dementia with Lewy bodies
Neurology:Question 41
A 35-year-old man comes to the office for evaluation of almost constant, involuntary movement of his hands
and, to a lesser extent, his feet. The movements have developed insidiously over the past 3 years. He denies
feeling nervous, and perceives that he has little control over the movements. He takes no medications and has
otherwise been in excellent health. His wife mentions recent personality change.
Abnormal involuntary movements are seen throughout the examination. There is no pattern to the movements.
His hands move continuously in rapidly flowing gyrations; he tends to sit on them when talking to prevent
them from moving. He walks with dancing movements that sometimes cause him to misstep. He has an
abnormal cognitive score on the office mental status examination.
Which of the following is the most likely cause of the movements?
A. Huntingtons chorea
B. Parkinsons disease
C. Creutzfeldt-Jakob disease
D. Tourettes syndrome
Neurology:Question 42
A 45-year-old woman with multiple sclerosis comes to the office as a new patient. Her disease began 15 years
ago with an attack of optic neuritis that resolved after 4 months. She was asymptomatic until 6 years ago, when
she had weakness of the right leg that responded to treatment with intravenous corticosteroids. Over the next 5
years she had a series of exacerbations that were less responsive to treatment with intravenous corticosteroids
and left her with residual neurologic deficits. In the past year, she has had gradual bilateral deterioration in leg
strength, such that she now requires a walker.
Which of the following best describes this patients course of multiple sclerosis?
A. Monosymptomatic demyelination
B. Relapsing-remitting
C. Primary progressive
D. Secondary progressive
E. Benign
Neurology:Question 43
A 75-year-old man comes to the emergency department because he has an ischemic stroke manifested by mild
expressive aphasia and a mild right hemiparesis especially involving his face and arm. He has a history of
hypertension, hypercholesterolemia, and cigarette smoking. Noninvasive cardiac evaluation reveals no
cardioembolic source. Carotid duplex ultrasonography shows 80% to 99% stenosis of the left internal carotid
artery (LICA) just above the carotid bifurcation. Catheter angiography reveals 70% stenosis of the extracranial
LICA and no other extracranial or intracranial stenosis.
Which of the following is the most appropriate management for this patient?
A. Intravenous heparin
B. Aspirin

C. Aspirin plus dipyridamole


D. Left carotid endarterectomy
E. Clopidogrel
Neurology:Question 44
A 75-year-old woman presents to the emergency department after a fall at home. She struck her head and was
briefly unconscious. She is now alert and fully oriented. Neurologic examination is normal. Computed
tomography of the head is negative for hemorrhage, but shows a 1-cm dural-based, calcifed, enhancing lesion
over her left cerebral convexity.
Which of the following is the most appropriate management of this patients lesion?
A. Gross total resection
B. Biopsy
C. Gamma-knife radiosurgery
D. Observation
Neurology:Question 45
A 72-year-old woman is admitted to the hospital after a cardiac arrest at home. Minutes after the event,
paramedics discern no heartbeat, but she regains sinus rhythm after a 30-minute resuscitative effort. An acute
myocardial infarction is diagnosed on admission. Five weeks after the cardiac arrest, she remains unresponsive
to voice and does not follow any commands. Her pupillary responses are intact, she has full lateral eye
movements to the dolls head maneuver, and she triggers the ventilator with spontaneous respirations. Her eyes
are open some of the time, and she appears to have sleep-wake cycles. There is no evidence of purposeful or
voluntary response to visual, auditory, tactile, or noxious stimuli, but she flexes her arms to noxious stimuli.
Which of the following is the most appropriate description of this patients neurologic condition?
A. Brain death
B. Locked-in syndrome
C. Minimally conscious state
D. Persistent vegetative state
Neurology:Question 46
A 29-year-old woman comes to the office because of gradual loss of vision in her left eye over 5 days. The
problem started as a smudge in her central visual field and gradually worsened such that she cannot read with
her left eye. She has pain with left eye movements, and she is finding it difficult to work because of the
impairment.
Neurologic examination is unremarkable except for eye findings. Visual acuity is 20/20 in the right eye and
20/400 in the left eye. Visual field testing shows a dense central scotoma on the left. Funduscopic examination
is normal. There is a left afferent pupillary defect. Magnetic resonance imaging of the brain is normal.
What is the best course of treatment?
A. Intravenous methylprednisolone
B. Interferon beta
C. Glatiramer acetate
D. Aspirin
E. Oral prednisone
Neurology:Question 47
A 26-year-old woman comes to the office with her husband, who provides the clinical history. According to him,
for the last couple of months the patient has been having episodes of sudden freezing and looking around,
during which she is unresponsive if people talk to her. The episodes last between 5 and 15 seconds, and occur 3
to 6 times a week. The patient does not recall these events and accuses her husband of making them up.
Physical examination is normal.
Which one of the following medications should be started while diagnostic work-up is in progress?
A. Lamotrigine
B. Phenobarbital
C. Gabapentin
D. Carbamazepine
E. Ethosuximide
Neurology:Question 48
A 55-year-old woman has noticed a tremor of her hands for the past 2 to 3 years. Initially, it was apparent when
she held a cup of coffee or wrote, but it now also interferes with her use of eating utensils and computer
keyboarding. She requests treatment. Medical and family history is negative and she takes no medications.
Examination is remarkable only for a moderately severe, bilateral postural and action tremor of both hands.
Routine blood studies, including thyroid-stimulating hormone, are normal.
Which of the following would be appropriate therapy?
A. Atenolol

B. Carbidopa/levodopa
C. Primidone
D. Valproic acid
E. Pramipexole
Neurology:Question 49
A 73-year-old woman is brought to the emergency department because of sudden onset of moderate right
hemiparesis and hemisensory deficit involving her face, arm, and leg equally. The patient has no relevant
medical history and takes no medications. She has no other signs or symptoms. She is admitted for inpatient
etiologic evaluation. Her average blood pressure during hospitalization is 138/82 mm Hg. Carotid duplex
ultrasonography shows nonstenotic plaque bilaterally. Transthoracic echocardiography shows diastolic
dysfunction but no potential cardioembolic source.
Which of the following should be recommended to the patient for blood pressure management?
A. Begin angiotensin-converting enzyme inhibitor in 2 weeks.
B. Begin oral -blocker immediately.
C. Begin oral calcium channel blocker immediately.
D. Recommend diet and lifestyle changes only.
Neurology:Question 50
A 37-year-old woman comes to the office because of recurrent headaches for 5 years. Initially, the headaches
were of moderate intensity and occurred approximately four times per year. In the past year, she has had rightsided severe, pulsatile pain associated with light and noise sensitivity and nausea without vomiting; the
headaches last 12 to 16 hours and occur once or twice monthly. The headaches cause her to miss work and other
activities occasionally. She uses acetaminophen without benefit. Physical and neurologic examinations are
normal.
Which of the following is most likely to be effective for acute treatment of this patients headaches?
A. Acetaminophen/ASA/caffeine combination
B. Acetaminophen/butalbital/caffeine combination
C. Aspirin
D. Naproxen
E. Zolmitriptan
Neurology:Question 51
A 30-year-old right-handed man with known HIV infection presents to the emergency department with a 3-day
history of progressive right hemiparesis and aphasia. Magnetic resonance imaging of the brain reveals a
homogeneously enhancing left frontal lesion. There is central necrosis and mass effect in the white matter of
the left cerebral hemisphere. A biopsy is performed and reveals diffuse B-cell lymphoma.
Genomic information from which of the following viruses is most likely to be found in this patients tumor cells?
A. Cytomegalovirus
B. Epstein-Barr virus
C. Herpes simplex virus
D. Herpes zoster virus
Neurology:Question 52
A 44-year-old woman with a history of hypertension presents to the emergency department after sudden onset
of the worst headache of her life. On examination, blood pressure is 190/95 mm Hg; pulse, 112/min ;
respiratory rate, 24/mm; and temperature, 36.8 C (98.2 F). She is mildly drowsy but arouses easily and is
cooperative. She has meningismus but no focal neurologic deficits. Computed tomography of the brain shows a
moderate amount of subarachnoid blood.
After basic medical care, which of the following is the most important acute treatment at this time?
A. Oral nimodipine
B. Intravenous labetalol
C. Intravenous dexamethasone
D. Intravenous enalapril
Neurology:Question 53
A previously healthy 38-year-old man has numbness in the right anterolateral leg that began 3 weeks ago and is
associated with a deep aching pain in the thigh. Ten days ago, he started to drag his right foot. Yesterday, he
became aware of numbness in the fourth and fifth digits of the left hand. He has lost 10 lb in the past month,
and has noted frequent night sweats. On examination, there is evidence of a right peroneal mononeuropathy
and left ulnar mononeuropathy. Serum chemistries, including fasting blood glucose level, are normal.
Which of the following is the most likely diagnosis?
A. Amyloid polyneuropathy
B. Vasculitic neuropathy

C. Guillain-Barr syndrome
D. Diabetic neuropathy
Neurology:Question 54
An 18-year-old college student comes to the emergency department because of a seizure that occurred at 8:30
this morning. According to her roommate, the patient woke up at her usual time and had several episodes of
brief arm jerks before she fell and lost consciousness. Her whole body stiffened for 5 to 10 seconds, and then
she had rhythmic clonic jerking of her arms, legs, and trunk for about 2 to 3 minutes. She remained
unresponsive for 10 minutes and then gradually regained complete consciousness over the next hour. The
patient remembers the arm jerks but has no recollection of the seizure. She states that she has been having arm
jerks in the morning for several months and that this has become worse recently. She has been staying up late
every night for the last week studying for final examinations. Neurologic examination is normal.
Which of the following is the most likely diagnosis?
A. Partial seizure with secondary generalization
B. Absence epilepsy
C. Syncope
D. Juvenile myoclonic epilepsy
E. Nonepileptic event (pseudoseizure)
Neurology:Question 55
A 26-year-old woman with clinically definite multiple sclerosis comes to the office because she is unexpectedly
pregnant and wants to discuss how to proceed with disease management. She is maintained on interferon beta
1a. She has had one acute exacerbation in the past 2 years. Neurologic examination shows right afferent
pupillary defect with mild weakness of the right leg and mild right-sided hyperreflexia. She asks how her
multiple sclerosis and treatment will affect the pregnancy.
Which of the following is the most appropriate counsel to share with her?
A. Continue interferon
B. Discontinue interferon beta
C. Change to glatiramer acetate
D. Recommend therapeutic abortion
Neurology:Question 56
A 42-year-old woman comes to the office because of a 3-year history of daily headaches that involve the left
retro-orbital, temporal, and parietal areas. The headaches begin when she awakens and persist all day. The pain
fluctuates from mild to moderate; at its most intense, she has left eye lacrimation and left-sided nasal
congestion and rhinorrhea. She occasionally has nausea. The patient takes propranolol 160 mg/d without
benefit. Physical and neurologic examinations are normal.
What is the most likely diagnosis?
A. Chronic tension-type headache
B. Chronic migraine
C. Cluster headache
D. Hemicrania continua

Neurology:Question 57
A 68-year-old woman comes for a routine physical examination. She describes a 2-year history of insomnia,
which she attributes to an inability to get comfortable in bed because of a creepy-crawly sensation in her
lower limbs. The sensation is absent during most of the day, but it has begun to bother her in the evening when
she sits and is especially troublesome in bed. Walking relieves the sensation. Physical examination, including
neurologic examination, is normal.
Which of the following studies should be done next?
A. Serum ceruloplasmin level
B. Serum ferritin level
C. Urine porphyrin level
D. Nerve conduction study
E. Magnetic resonance imaging of the brain

Neurology:Question 58
A 60-year-old woman is brought to the emergency department for evaluation of word-finding difficulty. She
called her husbands attention to the problem 3 weeks ago. Two weeks ago, the difficulty became apparent to
others, and she began to have difficulty finding her way around the offices at work. One week ago she had
difficulty navigating in her own home, confusing the garage, closet, and basement doors, and had difficulty with
arithmetic calculations and vision. Past medical and family history is unremarkable. On examination, she has
occasional myoclonic jerks of the upper extremities and facial musculature, worse on the right side. Her score is
15 of 30 on the Mini-Mental State Examination. Electroencephalography shows mild diffuse slowing. Magnetic
resonance imaging of the brain is normal.
What is the next step in testing?
A. Lumbar puncture
B. Presenilin-1 test
C. Positron emission tomography/single photon emission computed tomography
D. Repeat electroencephalography in 1 week
E. Brain and leptomeningeal biopsy
Neurology:Question 59
A 50-year-old previously healthy woman presents to her physician with a 3-week history of progressive gait and
limb ataxia, dysarthria, and blurred vision. Examination reveals nystagmus, dysarthria, and severe gait and
limb ataxia. The remainder of the neurologic examination is normal. Magnetic resonance imaging of the head
with contrast is normal. A blood test reveals positive serology for anti-Yo antibodies (anti-Purkinje cell
antibody).
Which of the following malignancies is most likely present in this patient?
A. Non-Hodgkins lymphoma
B. Small cell lung cancer
C. Ovarian cancer
D. Melanoma
Neurology:Question 60
A 34-year-old man comes to the office because of a 1-month history of episodes during which he is suddenly
unable to speak. The episodes last between 20 and 30 seconds. During some episodes, the patient has twitching
of the right side of his face; on one occasion the twitching progressed to involve his right arm. He has had five
events, the last three during the past week. No warning precedes the spells. The patient states that he is fully
aware of his surroundings during the episodes and has no other symptoms. Neurologic examination is normal.
Which of the following is the most likely diagnosis?
A. Transient ischemic attack
B. Anxiety attack
C. Frontal lobe seizure
D. Hemiplegic migraine
E. Hypoglycemia
Neurology:Question 61
A 24-year-old woman comes to the emergency department because of a 3-day history of severe headache. She
states that while she was at work she had sudden onset of excruciating, diffuse headache associated with nausea
and vomiting. She was unable to continue working, and a colleague drove her home. Over the next 2 days the
headache remained too severe for her to go to work. She has no significant past medical history except tobacco
use. On examination, blood pressure is 144/90 mm Hg; pulse, 104/min; respiratory rate, 18/mm; and
temperature, 37.2 C (98.9 F). She is mildly agitated and has photophobia and minimal neck stiffness. There are
no focal neurologic deficits. Computed tomography of the brain is normal.
What is the most appropriate next step in the management of this patient?
A. Emergent magnetic resonance imaging
B. Lumbar puncture
C. Cerebral arteriography
D. Treatment with sumatriptan
E. Treatment with ketorolac
Neurology:Question 62
A 35-year-old woman comes to the office because of episodic, severe headaches with left retro-orbital and
temporal pain and light and noise sensitivity; they are typically accompanied by severe nausea and vomiting
shortly after onset. The headaches most commonly awaken her from sleep in the early morning and last 16 to
24 hours. They have occurred approximately one or two times per month for the past 4 years. There are no
headache triggers. She has used over-the-counter medications and oral naratriptan without benefit. Physical
and neurologic examinations are normal.
Which of the following is the most appropriate acute treatment for this patients headaches?

A. Oral almotriptan
B. Oral rizatriptan
C. Intravenous dihydroergotamine
D. Subcutaneous sumatriptan
Neurology:Question 63
A 75-year-old man comes for an office visit because he is concerned about memory loss. He reports a family
history of Alzheimers disease, with dementia affecting three first-degree relatives in their 50s. His difficulties
are not apparent to his spouse or close friends, and have not affected his ability to perform his daily activities.
He is an active and successful financial officer. Past medical history is remarkable for treated hypertension and
osteoarthritis. There is no evidence of depression. Physical examination is normal. Mental status examination
discloses mild to moderate memory impairment and no abnormalities in other areas of cognition. Levels of
thyroid-stimulating hormone and vitamin B12 are normal, as is magnetic resonance imaging of the brain.
Which of the following is the most appropriate next step?
A. Presenilin-1 test
B. Follow-up evaluation in 1 year
C. Positron emission tomography/single photon emission computed tomography
D. Lumbar puncture
E. Diffusion-weighted magnetic resonance imaging
Neurology:Question 65
A 35-year-old woman comes to the office because she cannot prevent her neck from turning to the right. The
condition developed insidiously over 2 years and is very painful. She is the secretary for a small law firm and is
under pressure at work. She recalls no trauma and is otherwise in good health. On examination, she sits with
her head turned 45 degrees to the right; walking exacerbates the condition. She is unable to rotate her head to
the left. Cervical paraspinal muscles are tense. The remainder of the examination is normal.
Which of the following is most likely to be helpful?
A. Diazepam
B. Botulinum toxin injections
C. Carbidopa/levodopa
D. Cyclobenzaprine
E. Psychiatry referral
Neurology:Question 66
A 26-year-old woman comes to the office because of recurrent episodes of unusual behavior. She usually has a
warninga weird feeling in her stomach prior to the episodes. Her husband has witnessed most of the
events, and states that the patient suddenly freezes, looks around, smacks her lips, and moves her fingers
repetitively. The episodes usually last between 30 seconds and 2 minutes. She has no recollection of the events
after the warning. Afterward, the patient is confused and disoriented for as long as 10 minutes. The patient has
had at least 10 episodes over the last 2 years.
Which of the following is the most likely diagnosis?
A. Complex partial seizures
B. Absence seizures
C. Nonepileptic events (pseudoseizures)
D. Migraine
E. Transient global amnesia
Neurology:Question 67
A65-year-oldwoman presentsto herphysicianwith a 1-month historyof proximal muscleweakness, with difficulty
lifting her arms above her head and rising from a low chair. She also notes dry mouth and eyes and postural
light-headedness. She has recently completed chemotherapy for small cell lung cancer. Examination reveals
symmetric proximal muscle weakness that improves with repeated effort. Deep tendon reflexes are absent. The
remainder of the neurologic examination is normal. Blood pressure in the right arm is 125/80 mm Hg lying
down and 100/50 mm Hg standing. Pulse rate remains 70/min in either position.
Which of the following is the most likely diagnosis?
A. Metastatic spinal cord compression
B. Myasthenic syndrome
C. Polymyositis
D. Polyneuropathy
Neurology:Question 68
A 52-year-old woman comes to the emergency department because of a severe generalized, pulsatile headache
accompanied by light and noise sensitivity, nausea, and repeated vomiting. The headache began 16 hours
earlier. She has a 25-year history of recurrent migraine headaches. She takes amitriptyline for migraine

prophylaxis and has used oral rizatriptan today. Pulse rate is 120/min and regular; brachial blood pressure is
160/94 mm Hg. Physical and neurologic examinations are otherwise normal.
In addition to intravenous fluid and electrolyte replacement, what is the most appropriate next step in
management?
A. Intravenous dihydroergotamine
B. Intravenous dihydroergotamine and metoclopramide
C. Intravenous chlorpromazine
D. Oral codeine
E. Subcutaneous sumatriptan
Neurology:Question 69
A 73-year-old man has sudden onset of right hem iparesis and expressive aphasia that improves nearly to
baseline by the time he arrives in the emergency department. He is admitted to the hospital for rapid etiologic
evaluation. Laboratory studies are normal except for a low-density lipoprotein level of 110 mg/dL. Carotid
duplex ultrasonography shows left carotid stenosis of 50%.
Which of the following is the most appropriate management for this patients serum cholesterol?
A. Begin pravastatin.
B. Begin niacin.
C. Recommend diet and lifestyle changes only.
D. Recommend no further intervention.
Neurology:Question 70
A 25-year-old man with a 15-year history of seizures comes to the office for routine follow-up. He has never
been seizure free for more than a few weeks. A feeling of dj vu precedes the seizures. He then has lip
smacking and repetitive movements of his left fingers that continue for less than a minute and are followed by a
generalized tonic-clonic seizure. He has had many injuries due to seizures. The patient has tried several
combinations of antiepileptic drugs with varying success. Currently, he takes a combination of valproic acid and
lamotrigine. Serum levels are in the therapeutic range. The patient has had several electroencephalograms
showing focal right temporal lobe epileptiform activity. Magnetic resonance imaging of the brain performed 6
months ago is consistent with right mesial temporal sclerosis.
Which of the following is the most appropriate next management step?
A. Repeat magnetic resonance imaging of the brain
B. Refer for consideration of temporal lobe surgery
C. Order positron emission tomography
D. Treat with vagal nerve stimulation
E. Add a third antiepileptic drug
Neurology:Question 72
A 27-year-old man comes to the office for advice about treatment of his recently diagnosed multiple sclerosis.
Three years ago he had an episode of diplopia that resolved entirely after 2 months. Six months ago, he had
mild weakness and numbness of the right leg. Magnetic resonance imaging of the brain at that time showed
multiple cerebral white matter lesions in a periventricular distribution classic for multiple sclerosis. His leg
weakness resolved without treatment. He is now asymptomatic and has a normal neurologic examination.
Which of the following is the best treatment recommendation at this point?
A. High-dose intravenous methylprednisolone
B. Chronic oral prednisone
C. Azathioprine
D. Interferon beta
E. Observation
Neurology:Question 73
A 25-year-old woman comes to the office because of a 4-month history of generalized, nonpulsatile headaches
of moderate severity. The headaches are associated with intermittent nausea and episodes during which the
vision in her right eye fades for several seconds. She takes naproxen sodium 3 days per week. Physical
examination is normal exceptfora body mass index of 32.1. Neurologic examination shows bilateral swelling of
the optic discs. Magnetic resonance imaging of the brain and magnetic resonance venography are normal.
Examination of CSF shows an opening pressure of 35 cm H2O, total protein of 35 mg/dL, glucose of 64 mg/dL,
and 2 leukocytes/L; the fluid is negative for VDRL. What is the most likely diagnosis?
A. Hydrocephalus
B. Chronic migraine
C. Borrelia burgdorferi meningitis
D. Idiopathic intracranial hypertension
E. Medication overuse headache

Neurology:Question 74
A 72-year-old woman is brought to the office by her daughter for evaluation of mental status changes. The
patient had a stroke 1 month earlier, with mild residual left-sided weakness. She had one seizure while in the
hospital; the seizure was considered grand mal, and she was treated with phenytoin. According to the daughter,
the patients left face and arm twitch frequently, and her left side is weaker than when she left the hospital. For
the last few days the patient has been confused and disoriented, like in a daze. Her current medications
include phenytoin and aspirin. On examination, vital signs are normal. The patient is somewhat lethargic and
has frequent twitching of her left face and arm. There is left facial weakness and left hemiparesis involving the
face, arm, and leg. Complete blood count, serum electrolytes, and urinalysis are normal.
Which of the following is the most likely diagnosis?
A. Stroke in evolution
B. Partial status epilepticus
C. Phenytoin toxicity
D. Creutzfeldt-Jakob disease
Neurology:Question 75
A 78-year-old man is brought to the office by his family because of memory decline and confusion. The family
reports progressive confusion over 6 months. The family noticed his symptoms at first mainly in the morning,
but they now seem to occur throughout the day. He does not recognize his spouse at times, and thinks someone
is chasing him. He wanders from the house, has visual hallucinations, and has difficulty using household
appliances. Medical history is significant for diabetes with painful peripheral neuropathy, coronary artery
disease, and congestive heart failure. Medications include glyburide, nortriptyline, digoxin, lorazepam,
diltiazem, and lisinopril. There is no family history of neurologic disease. On examination, the patient has mild
asterixis and findings consistent with peripheral neuropathy. He is mildly lethargic and inattentive, and not
oriented to time or place. He recalls two of three words after a delay. The Mini-Mental State Examination score
is 13 of 30. Electrolyte levels, oxygen saturation, liver function, and renal function are normal. Computed
tomography of the head without contrast is normal.
Which of the following conditions is most likely to account for his cognitive impairment?
A. Adverse effect of medication
B. Dementia with Lewy bodies
C. Cerebrovascular disease
D. Depression
ANSWERS
ANSWERS
Neurology:Question 1
The correct answer is B
Educational Objectives
Manage a patient with a single seizure.
Critique
Most neurologists would not start antiepileptic drug therapy after a single seizure unless the patient has one or
more risk factors associated with seizure recurrence: abnormal neurologic examination, abnormal
electroencephalography or magnetic resonance imaging of the brain, partial onset, a history of neurologic
injury (such as stroke, hemorrhage, or tumor), family history of seizures, or postictal motor paralysis. This
patient has no risk factor for recurrence of seizures. Her seizure was provoked by sleep deprivation. Seizures
caused by external events (drug toxicity, drug withdrawal, sleep deprivation, hypoglycemia, hypoxia,
hyponatremia) are considered provoked and do not require chronic antiepileptic drug treatment. Patients with
a single provoked seizure should have electroencephalography and magnetic resonance imaging to rule out
underlying focal pathology that could lower the seizure threshold and precipitate a seizure. Not ordering any
tests and just treating the patient with an antiepileptic drug can lead to missing an early diagnosis of brain
tumor, small stroke, hemorrhage, or other focal cortical abnormality.
Neurology:Question 2
The correct answer is E
Educational Objectives
Initiate urgent workup for a patient with intermittent atrial fibrillation and transient ischemic attack.
Critique
Patients with transient ischemic attacks (TIA5) should be evaluated and treated urgently. Among patients who
come to the emergency department with TIA, 5% will have a stroke within 2 days, and 25% will have a recurrent
event within 3 months. Patients with atrial fibrillation and TIA or stroke should receive life-long oral
anticoagulation to prevent cerebral ischemia from cardioembolism. Emergency computed tomography should
be done in this patient to rule out tumor or hemorrhage as the cause of hemiparesis. Once bleeding is ruled out,

anticoagulation should be initiated urgently. Warfarin benefits patients with chronic or intermittent atrial
fibrillation regardless of left atrium size, so findings on Holter monitoring and transesophageal
echocardiography would not affect initial management. It is appropriate to obtain carotid duplex
ultrasonography, since 5% to 10% of patients with atrial fibrillation have concurrent carotid stenosis, but this is
less immediately urgent.
Neurology:Question 3
The correct answer is D
Educational Objectives
Monitor disease activity in a patient with a monosymptomatic demyelinating event.
Critique
According to the new McDonald criteria for diagnosis of multiple sclerosis, new disease activity demonstrable
by new T2 white matter lesions, or new gadolinium-enhancing lesions on serial brain or spinal cord magnetic
resonance imaging (MRI) at least 3 months after an initial scan qualify as dissemination of demyelination
across time. Serial MRI is the best way to monitor for ongoing disease activity. Much of the activity of multiple
sclerosis is subclinical and shows up only on MRI without changes in neurologic examination or new
symptoms. Monitoring of visual system function would not help because new disease activity must be
disseminated across space (in multiple areas of the central nervous system) and the optic nerves have already
been affected. Nerve conduction velocities test for abnormalities in the peripheral nervous system, not the
central nervous system, which is affected by multiple sclerosis.
Neurology:Question 5
The correct answer is B
Educational Objectives
Recognize medication overuse as a cause of chronic daily headache.
Critique
This patient has developed a form of daily headache known as chronic migraine (transformed migraine). In
many patients the change in headache pattern is caused by overuse of analgesics (more than 2 to 3 days per
week). Combination analgesics are particularly likely to lead to chronic daily headache. Accordingly, the first
intervention is to discontinue use of the combination analgesic. Discontinuing the offending agent usually
results in improvement within 1 month, but occasionally takes longer. Patients should be advised that
headaches may transiently worsen when the medication is discontinued. Amitriptyline may prove to be an
effective preventive agent once the medication overuse headache has been eliminated. Increased amitriptyline
or substitution with divalproex sodium is not likely to improve the daily headaches as long as the overuse of
analgesics persists. Overuse headache from triptan agents is unlikely to occur when the medication is taken at a
frequency of 2 days per week, as in this patient.
Neurology:Question 6
The correct answer is D
Educational Objectives
Recognize the clinical presentation of amyotrophic lateral sclerosis.
Critique
This patient has progressive, painless weakness and atrophy in the absence of sensory symptoms. He has upper
and lower motor neuron signs in the symptomatic left arm, and essentially asymptomatic weakness in the left
leg. The presentation suggests motor neuron disease (amyotrophic sclerosis). EMG would most likely confirm
the diagnosis with findings of denervation in multiple extremities. As only 5% to 10% of cases of amyotrophic
lateral sclerosis are familial, genetic testing at this stage is not indicated. The distribution of weakness is not
proximal, as would occur with most myopathies, so muscle biopsy would not be helpful. Although creatine
kinase levels may be elevated in rapidly progressive motor neuron disease, it is a nonspecific finding. The
absence of sensory symptoms or signs argues against a peripheral neuropathy, so nerve biopsy, while likely
showing axonal loss, would not be of high yield.
Neurology:Question 7
The correct answer is C
Educational Objectives
Manage incidental unruptured aneurysm.
Critique
This patient has a 4-mm asymptomatic intracranial aneurysm. The natural history of asymptomatic aneurysms
less than 10 mm in diameter is usually benign, with a very low risk of rupture. In general, these patients should
probably not undergo surgical therapy unless the aneurysm enlarges or changes shape. Periodic noninvasive
testing is appropriate. Computed tomographic angiography has little to add to the magnetic resonance
angiography that has already been performed in this patient. Catheter angiography is associated with additional
risk and does not improve assessment of asymptomatic aneurysm.

Neurology:Question 8
The correct answer is D
Educational Objectives
Initiate treatment of symptomatic Parkinsons disease.
Critique
This man has signs and symptoms of parkinsonism, and very likely Parkinsons disease, that are causing him to
restrict his lifestyle. Treatment is indicated to keep him engaged in activities appropriate for his age.
Carbidopa/levodopa is the appropriate first choice of medication. Dopamine agonist medications, such as
pramipexole, ropinirole, pergolide, or bromocriptine, are not as effective as carbidopa/levodopa and may be
insufficient to counter his increasing parkinsonian symptoms. Selegiline and amantadine are unlikely to have a
major impact upon his symptoms. Entacapone is ineffective in the absence of levodopa treatment and hence
has no role in initial treatment.
Neurology:Question 9
The correct answer is A
Educational Objectives
Recognize Alzheimers disease.
Critique
This patients progressive cognitive impairment affects both multiple aspects of intellect and his ability to
perform daily tasks. The findings are consistent with a progressive dementia. Prominent recent memory loss
suggests early involvement of the temporal lobes. The lack of other neurologic signs suggests sparing of the
basal ganglia and primary motor and sensory areas. The findings are most compatible with dementia due to
Alzheimers disease. Frontotemporal dementia may cause a similar profile, but usually has more prominent
early personality change and less visuospatial impairment. Imaging often shows focal frontotemporal brain
atrophy. Dementia with Lewy bodies is accompanied by parkinsonism, hallucinations, and fluctuating
symptoms, none of which is present here. Creutzfeldt-Jakob disease has a much more rapid course and is
usually associated with motor signs such as rigidity, ataxia, or myoclonus. Vascular dementia is unlikely in the
absence of vascular risk factors, focal abnormalities on imaging, or a stroke-like (abrupt or stepwise) course of
illness.
Neurology:Question 10
The correct answer is B
Educational Objectives
Recognize the clinical presentation of nonepileptic events (pseudoseizures).
Critique
This is a typical presentation of a nonepileptic event (pseudoseizure). Pseudoseizures usually start in an
emotionally charged setting, with a gradual buildup of symptoms. Symptoms and duration are not
stereotypical. The movements are not synchronous and are usually prolonged, starting and stopping many
times throughout the event. Women are more likely than men to have nonepileptic events; associated factors
include health care occupation, major life stressors, medical history of unexplained physical symptoms,
psychiatric comorbidity, and traumatic childhood, including history of abuse. Many patients with nonepileptic
episodes have family members or friends with seizures or have witnessed a seizure.
Syncope often causes tonic posturing and myoclonic jerks, but the episodes are brief and there is fast and full
recovery of function (no postictal confusion). True generalized tonic-clonic seizures are characterized by tonic
extension of the whole body, followed by synchronous, rhythmic clonic movements of the limbs. Partial seizures
with secondary generalization have a focal onset. Basilar migraine is characterized by recurrent stereotypical
episodes of transient brainstem symptoms such as vertigo or diplopia, and is usually associated with headache.
Neurology:Question 11
The correct answer is E
Educational Objectives
Initiate etiologic evaluation to determine appropriate therapy for secondary stroke prevention.
Critique
The choice of antithrombotic agent for secondary stroke prevention depends on the cause of stroke; initiating
therapy without taking into account the cause of stroke is not appropriate. Patients with atrial fibrillation
benefit most from chronic anticoagulation with warfarin (target INR [international normalized ratio] between
2.0 and 3.0). In patients with large- or small-artery disease, antiplatelet therapy decreases the risk of stroke and
death, with no additional benefit from warfarin.
Neurology:Question 12
The correct answer is C
Educational Objectives
Recognize the locked-in syndrome that can occur due to acute pontine lesions.
Critique

This patient is not comatose, but is in a locked-in state. Patients in a locked-in state are quadriplegic, have
paralysis of horizontal eye movements, and are unable to speak because of paralysis of bulbar muscles;
communication is possible only through the patients ability to move the eyes vertically and blink. The locked-in
syndrome occurs with lesions of the base of the pons, usually infarction (such as from basilar artery occlusion)
or hemorrhage. The finding of brisk downward eye movements is consistent with ocular bobbing, a kind of
spontaneous eye movement that can be seen in large pontine lesions. Although not entirely specific, another
clue to a brainstem localization of this patients findings is the presence of bilateral extensor (decerebrate)
posturing. The bilaterally small pupils are also commonly associated with pontine lesions. A left middle
cerebral artery stroke would cause aphasia and a right hemiparesis, not the bilateral signs or the eye findings
seen in this patient. An anoxic encephalopathy would cause diffuse cerebral, and sometimes brainstem,
dysfunction but would not explain this patients eye movements or the intact alertness in the presence of
quadriplegia. The typical findings of a cerebellar infarct are headache, nausea, vomiting, and ataxia; brainstem
(including pontine) dysfunction can occur as a secondary phenomenon due to mass effect from infarct-related
edema, which is why neurosurgical consultation is important for large cerebellar infarcts as well as for
cerebellar hemorrhages.
Neurology:Question 13
The correct answer is A
Educational Objectives
Recognize effect of fever on multiple sclerosis.
Critique
Neurologic function often deteriorates when patients with multiple sclerosis are exposed to ambient heat or
fever, because raised temperature decreases conduction through demyelinated nerves. This is considered a
pseudoexacerbation and is not necessarily evidence of a new exacerbation. Sudden worsening of lowerextremity strength and increased spasticity in any patient with multiple sclerosis, especially if associated with
fever, should prompt investigation for urinary tract infection, decubitus ulcer, cellulitis, arthritis, or other
irritative conditions. Any painful or irritating stimulus can worsen spasticity. Often the course improves over
several days with hydration, appropriate antibiotic therapy, and aggressive reduction of fever. The other
treatment options will not directly address this patients deterioration due to fever.
Neurology:Question 14
The correct answer is D
Educational Objectives
Recognize the clinical features of migraine headaches.
Critique
This patients symptoms fulfill the criteria for migraine without aura, despite the bilateral pain and absence of
nausea. Rhinorrhea and tearing are autonomic symptoms that frequently accompany migraine and do not
usually represent sinus disease. Most patients with a diagnosis of sinus headache in fact have migraine. Acute
sinusitis is accompanied by fever and purulent nasal discharge. Episodic tension-type headaches are not
pulsatile, do not worsen with physical activity, and are not associated with photophobia and phonophobia.
Cluster headaches are unilateral (most commonly retro-orbital), last 1 to 2 hours, and occur in clusters of
recurrent attacks lasting weeks to months. Most patients are agitated and prefer not to lie down during a cluster
headache. This patient has no signs or symptoms, such as visual obscurations, that suggest a diagnosis of
idiopathic intracranial hypertension, and there is no papilledema on funduscopic examination.
Neurology:Question 15
The correct answer is A
Educational Objectives
Recognize brain metastases.
Critique
This patients history of node-positive breast cancer places her at high risk for development of recurrent and
metastatic tumor. She presents with progressive signs and symptoms of focal neurologic deficit and raised
intracranial pressure. She most likely has multiple brain metastases. Magnetic resonance imaging of the head
with contrast would be the most appropriate diagnostic test for this patient. Parenchymal brain metastases are
by far the most common neurologic complication of systemic cancer, occurring in 20% to 40% of patients.
Leptomeningeal and subdural metastases are less common. Patients with leptomeningeal metastases typically
present with headache, multiple cranial nerve palsies, and multiple spinal radiculopathies. Patients with
subdural metastases and dural sinus thrombosis primarily present with headache, hemiparesis, and sometimes
drowsiness. Hemianopia would be distinctly unusual with a lesion not directly involving brain parenchyma.
Paraneoplastic encephalitis is rare. It presents with seizures and rapidly progressive cognitive and personality
changes; it is not usually associated with headache or signs of raised intracranial pressure. A cerebral infarct
presents as an acute-onset neurologic deficit without symptoms or signs of raised intracranial pressure.

Neurology:Question 16
The correct answer is A
Educational Objectives
Manage antiepileptic drug therapy in a pregnant patient.
Critique
During pregnancy, seizure frequency increases in approximately one third of women with epilepsy. This is a
result of increased hepatic metabolism of most antiepileptic drugs and an increased volume of distribution.
Therefore, drug levels should be monitored during pregnancy to make sure they remain therapeutic. This
patient appears to have good seizure control on the current dose of carbamazepine. There is no need to change
the dose or add or switch to another antiepileptic drug at the present time. If the patient starts to have
breakthrough seizures, the dose of carbamazepine should be increased instead of adding or changing to another
antiepileptic drug. Changing antiepileptic drug therapy during pregnancy can be harmful because during the
transition from one drug to another there is a risk of toxicity or breakthrough seizures. Ideally, the patient
would have discussed plans to become pregnant in advance, so any necessary changes to her antiepileptic drug
regimen could be made before the patient conceives. Since often pregnancies are not planned, every woman of
childbearing age should take folate supplements to reduce the risk of neural tube defects from anticonvulsant
therapy.
The goal is no seizures on the minimal effective dose of a single antiepileptic drug. Combination of antiepileptic
drugs should be avoided if possible since there is an increased risk of congenital malformations and side effects.
For many women, withdrawal of antiepileptic drugs before pregnancy results in recurrence or exacerbation of
seizures, which can be dangerous for both mother and fetus.
Neurology:Question 17
The correct answer is C
Educational Objectives
Recognize cerebellar hemorrhage.
Critique
This patient has classic signs and symptoms of an acute left cerebellar insult, including headache, nausea,
vomiting, vertigo, and left-sided ataxia. The etiology is either cerebellar hemorrhage or cerebellar infarction. He
requires emergent computed tomography to check for cerebellar hemorrhage; if hemorrhage is present, he
would need urgent neurosurgical consult. Cerebellar hemorrhages larger than 3 cm or with mass effect on the
brainstem usually require decompression, which is often lifesaving, and patients can recover with minimal
neurologic deficits. The other options are peripheral vestibular disorders that are unlikely to cause headache or
the cerebellar findings in this patient.
Neurology:Question 18
The correct answer is B
Educational Objectives
Diagnose chronic inflammatory demyelinating polyneuropathy.
Critique
The diagnostic features of chronic inflammatory demyelinating polyneuropathy include roughly symmetric
weakness and sensory loss progressing over at least 2 months, with evidence of segmental demyelination on
nerve conduction studies. Although results of electrodiagnostic testing are not given in this case, chronic
inflammatory demyelinating polyneuropathy is the most likely diagnosis of the options given. Polymyositis and
myasthenia gravis would not produce sensory signs or symptoms. The progression of symptoms over 8 months
is too long for Guillain-Barr syndrome, which evolves over 4 to 6 weeks.
Neurology:Question 19
The correct answer is C
Educational Objectives
Treat patient with acute ischemic stroke and persistently elevated blood pressure.
Critique
Management of blood pressure is especially complex in patients with acute ischemic stroke who are otherwise
eligible for intravenous tissue plasminogen activator (t-PA). In stroke patients who receive intravenous t-PA,
elevated blood pressure carries an increased risk for intracranial hemorrhage, hence persistent systolic blood
pressure above 185 mm Hg or persistent diastolic blood pressure above 110 mm Hg is a contraindication to
intravenous t-PA. However, acute hypertension at the time of stroke is most often a result ofnot a cause of
stroke, and is necessary to supply the reversibly ischemic brain (ischemic penumbra) with enough blood to
survive. Since the goal of intravenous t-PA therapy is to save the penumbra, it is counterproductive to lower
blood pressure excessively in order to administer t-PA. Thus, the need for aggressive therapy to achieve blood
pressure below 185/110 mm Hg is also a contraindication to intravenous t-PA. In this patient, the systolic value
would need to be lowered by 55 mm Hg in order to administer intravenous t-PA within the next hour. This is
considered aggressive therapy.

Intra-arterial t-PA is not currently approved by the U.S. Food and Drug Administration for ischemic stroke; at
this time, it is used in specialized centers for patients who present within 3 to 6 hours of onset of symptoms.
Recommendations regarding blood pressure management are the same for intra-arterial and intravenous t-PA.
Neurology:Question 20
The correct answer is B
Educational Objectives
Manage prophylaxis for migraine headaches.
Critique
The patient is tolerating the amitriptyline well; since the dose is currently low, it is appropriate to continue to
increase it gradually as planned. Maximal benefit of a prophylactic agent may not be recognized for 2 to 3
months after attaining the target dose. Accordingly, it would be an error to continue with the current dose or to
discontinue amitriptyline. Propranolol and divalproex sodium are both useful prophylactic agents, but should
not be substituted for amitriptyline at this time. They might be considered if amitriptyline is ultimately not
effective or tolerated.
Neurology:Question 21
The correct answer is A
Educational Objectives
Recognize that antiemetic drugs with dopamine antagonist properties may induce parkinsonism.
Critique
This patient has neurologic symptoms consistent with parkinsonism. Metoclopramide, a commonly prescribed
antiemetic that blocks dopamine receptors, may cause parkinsonism and could be responsible for his
symptoms.
Gluten sensitivity has been associated with peripheral neuropathies and cerebellar ataxia, but is not a likely
cause of parkinsonism. Neither ranitidine nor bupropion has been associated with development of
parkinsonism. Selective serotonin reuptake inhibitors (SSRI5) have been reported to cause parkinsonism, but
this is extremely rare; they are not contraindicated and usually are well tolerated by patients with Parkinsons
disease. Symptomatic treatment with carbidopa/levodopa or pramipexole may be appropriate later, if
discontinuation of metoclopramide fails to resolve parkinsonism.
Neurology:Question 22
The correct answer is D
Educational Objectives
Recognize leptomeningeal metastases.
Critique
This patient has known lung cancer and presents with subacute multifocal neurologic deficits affecting multiple
levels of the nervous system (ventricular system, cranial nerves, lumbar root). This is the classic presentation of
leptomeningeal tumor, which occurs in 3% to 8% of all patients with cancer. Communicating hydrocephalus is
common and due to obstruction of the arachnoid granulations. Multifocal cranial neuropathies and
radiculopathies are frequent and due to nerve infiltration by tumor cells within the cerebrospinal fluid.
Multiple parenchymal metastases could produce a similar clinical picture but generally would be manifest on
computed tomography. Paraneoplastic neuropathy and herpes zoster infection of the central nervous system
are extremely rare complications of cancer; paraneoplastic neuropathy would not cause cranial nerve deficits.
Venous sinus thrombosis would not produce multifocal cranial neuropathy and lumbar radiculopathy.
Neurology:Question 23
The correct answer is C
Educational Objectives
Distinguish seizures from syncope.
Critique
This patients symptoms are most compatible with vasovagal syncope. There is usually a brief prodrome that
may include lightheadedness, warmth, diaphoresis, nausea, pallor, and blurred vision. Abnormal movements
(mainly myoclonic jerks or tonic posturing) and urinary incontinence may occur during unconsciousness. Once
recumbent, the patient usually recovers consciousness rapidly but may be confused for up to 30 seconds.
Juvenile myoclonic epilepsy is a genetic syndrome characterized by myoclonic jerks and absence and
generalized tonic-clonic seizures. Affected patients have no warning prior to the generalized tonic-clonic
seizure. Afterward, the patient falls into a deep sleep or is tired and confused for minutes to hours. Complex
partial seizures with secondary generalization are associated with auras, but patients are not likely to be pale or
diaphoretic or to feel warm. Postictal confusion lasts minutes to hours. Pseudoseizures are not stereotypical in
presentation and usually last longer. Atonic seizures consist of sudden loss of postural tone without any
prodrome or warning and without loss of consciousness.
Neurology:Question 24
The correct answer is C

Educational Objectives
Treat pathologic fatigue of multiple sclerosis.
Critique
Amantadine is the first-line pharmacologic agent for treatment of multiple sclerosis fatigue, which is common
and disabling. Amantadine affects several neurotransmitter systems and enhances neurotransmitter release;
however, the exact mechanism by which it improves fatigue of multiple sclerosis is not known. It is well
tolerated in younger patients with multiple sclerosis despite anticholinergic effects at doses such as 100 mg
orally each morning or twice a day (morning and afternoon).
Other causes of severe fatigue should be sought in all patients with multiple sclerosis. Typically, patients should
be screened for thyroid dysfunction, but thyroid supplementation is used only if laboratory abnormalities
suggest thyroid disease. Amphetamines and agents such as ritalin are not first-line therapy because of their
addiction potential.
No firm data support use of selective serotonin reuptake inhibitors for fatigue of multiple sclerosis. One study
suggests an overlap between depression and fatigue in multiple sclerosis, but they are believed to be due to
different mechanisms. Pemoline has been used for multiple sclerosis fatigue in the past, but it is now used
infrequently because frequent liver function tests are needed to check for hepatotoxicity. Modafinil, a
nonamphetamine stimulant used extensively in narcolepsy, is useful in fatigue of multiple sclerosis as well. Its
primary limitation is cost. Other approaches to fatigue include energy conservation, graded exercise programs
to promote stamina, and elimination of medications that may contribute to sedation.
Neurology:Question 25
The correct answer is A
Educational Objectives
Use cholinergic augmentation in Alzheimers disease.
Critique
This patient has symptoms consistent with progressive dementia, most likely Alzheimers dementia. Clinical
trials consistently demonstrate modest efficacy of cholinesterase inhibitors, including galantamine, tacrine,
donepezil, and rivastigmine, in mild to moderate Alzheimers disease, on both cognitive and global function
scales. In addition, cholinesterase inhibitors act favorably on psychiatric symptoms, including agitation, apathy,
and hallucinations. There is no documented efficacy of these drugs in delaying the natural history of
Alzheimers disease. Gingko biloba probably exerts modest effects on cognitive symptoms, but has not been
shown to improve global or psychiatric function. Although some studies suggest that vitamin E and selegiline
delay the progression of Alzheimers disease, they do not directly relieve cognitive or psychiatric symptoms.
Neurology:Question 26
The correct answer is A
Educational Objectives
Administer intravenous tissue plasminogen activator (t-PA)to ischemic stroke patients only within 3
hours of stroke onset.
Critique
Intravenous t-PA decreases the risk of disability in patients with acute ischemic stroke only when given
according to the protocol established in 1995 in studies by the National Institute of Neurological Disorders and
Stroke (NINDS). Chief among the inclusion criteria is stroke onset within 3 hours of when the patient was last
known to be normal. Since patients with acute stroke often cannot give accurate histories, the histories
obtained from witnesses and caregivers are essential. According to protocol criteria, this patients stroke onset
was 3 days earlier. The computed tomography findings lend further evidence that the stroke was not acute: a
hypodense (dark) area with mass effect suggests a subacute (days-old) infarction.
Corticosteroids are not indicated and may be harmful in patients with ischemic stroke. The cytotoxic cerebral
edema that occurs with ischemic stroke is greatest 2 to 5 days after stroke onset, and does not respond to
corticosteroids. Corticosteroids are effective for vasogenic edema due to mass lesions such as tumors. Despite
its frequent use in the United States, there is no evidence that intravenous heparin is safe and effective in
patients with acute ischemic stroke; in fact, this therapy carries increased risk of hemorrhagic transformation of
the infarction, especially in large subacute infarctions. Intra-arterial t-PA is not currently approved by the U.S.
Food and Drug Administration for ischemic stroke; at this time, it is used in specialized centers for patients
who present within 3 to 6 hours of onset of symptoms.
Neurology:Question 27
The correct answer is E
Educational Objectives
Recognize multiple system atrophy.
Critique
This patient has evidence of dysautonomia (with orthostatic hypotension and possible neurogenic bladder),
parkinsonism, corticospinal tract signs, and ataxia. The constellation of findings is typical of multiple system

atrophy. The screeching sounds during the night are worrisome for nocturnal stridor, which is highly
suggestive of multiple system atrophy. Polysomnography should be obtained to confirm nocturnal stridor
because patients with multiple system atrophy and stridor may die during the night, presumably from
dysfunction of the brainstem breathing center. Stridor can be treated with tracheostomy or possibly with
continuous positive airway pressure. Although Parkinsons disease is often associated with dysautonomia, the
pronounced autonomic symptoms, including symptomatic orthostatic hypotension, are atypical. Further,
corticospinal tract signs and ataxia are not expected in Parkinsons disease. Creutzfeldt-Jakob disease is
unlikely, given the clinical picture and the 2-year course.
Although spinocerebellar degenerations may manifest with parkinsonism, corticospinal tract signs, and ataxia,
they do not include prominent dysautonomia. Progressive supranuclear palsy manifests with parkinsonism and
often corticospinal tract signs, but not with severe dysautonomia with symptomatic orthostatic hypotension.
Also, the patient does not have the vertical (downward) gaze palsy suggesting progressive supranuclear palsy.
Neurology:Question 28
The correct answer is C
Educational Objectives
Recognize vitamin B12 deficiency.
Critique
This man has signs and symptoms of progressive spinal cord dysfunction. He has dense posterior column
dysfunction (vibration and proprioception) as well as corticospinal tract motor signs and deficits. This
combination suggests subacute combined degeneration, as seen in vitamin B12 deficiency. The normal
magnetic resonance image of the spinal cord rules out cord compression. However, other causes of progressive
myelopathy should be considered.
The other options are not upper motor neuron syndromes and are not associated with hyperreflexia or extensor
plantar responses.
Neurology:Question 29
The correct answer is D
Educational Objectives
Evaluate intracranial hemorrhage.
Critique
This patient has an intracerebral hemorrhage with intraventricular extension. Although the location of the
hemorrhage is typical for a hypertensive bleed, patients younger than 45 years who have no prior history of
hypertension are at much higher risk for a potential vascular anomaly underlying the hemorrhage. Cocaine use
could be solely responsible for the stroke; however, patients with cocaine-related hemorrhage are also more
likely to have a vascular anomaly. This patient needs aggressive evaluation for a possible underlying vascular
malformation, and the definitive test is cerebral angiography. Electroencephalography is not indicated unless
the history raises the possibility of seizure. Echocardiography would provide information about the possibility
of chronic hypertension in this patient, but patients younger than 45 years should have vascular imaging after
intracranial hemorrhage, even if they have known hypertension.
Neurology:Question 30
The correct answer is B
Educational Objectives
Diagnose acute diabetic proximal neuropathy.
Critique
This patients signs and symptoms are consistent with a lesion of the right upper lumbar root or right lumbar
plexus. The normal findings on magnetic resonance imaging argue against a structural or compressive cause.
The most likely noncompressive cause of right upper lumbar radiculoneuropathy is diabetes. Diabetic proximal
neuropathy (diabetic amyotrophy) may be symmetric or asymmetric, and presents with subacute proximal leg
muscle weakness associated with severe pain. Symptoms usually begin after longstanding uncontrolled
hyperglycemia and weight loss. The pathogenesis is unclear; nerve biopsy studies have shown a vasculitic
inflammatory infiltrate in involved nerves.
Obtaining a creatine kinase level would be appropriate if primary muscle disease were suspected, but this is not
likely given the presence of neuropathic pain and sensory loss. Thyroid function tests are often obtained in
cases of peripheral neuropathy, but thyroid derangement would not cause a lumbar radiculopathy or
plexopathy. Lumbar puncture may help confirm an inflammatory process in the cerebrospinal fluid, but would
not result in a specific diagnosis. Similarly, elevations of the erythrocyte sedimentation rate are nonspecific.
Neurology:Question 31
The correct answer is D
Educational Objectives
Diagnose progressive supranuclear palsy.
Critique

This patients early falls and vertical gaze paresis make progressive supranuclear palsy the likely diagnosis.
Patients with progressive supranuclear palsy often have vision complaints, yet their optometrists are unable to
document problems with either visual acuity or peripheral vision. Impaired eye movements, especially the
inability to look down, underlie these complaints. Many routine tasks, such as reading and needlework, require
down gaze and visual tracking.
Patients with idiopathic Parkinsons disease do not fall early in the course of disease. Although there are clues
that might suggest dystonia, multiple system atrophy, or cerebellar degeneration, the overall clinical picture fits
best with progressive supranuclear palsy.
Neurology:Question 32
The correct answer is D
Educational Objectives
Recognize frontotemporal dementia.
Critique
This patients progressive impairment of cognition with prominent personality and behavior changes is most
consistent with frontotemporal dementia. Brain imaging would be appropriate to exclude a mass lesion. Brain
imaging in patients with frontotemporal dementia typically shows pronounced atrophy of the frontal and
temporal lobes. Despite the patients high score on the Mini-Mental State Examination (MMSE), a diagnosis of
mild cognitive impairment is inappropriate, since the patient is clearly impaired in his interpersonal and
occupational function, and not only in memory. In fact, the paradoxically high score is a clue to the diagnosis,
because the MMSE is relatively insensitive to executive dysfunction, the predominant impairment in
frontotemporal dementia. The positive family history suggests an autosomal-dominant disorder, which is more
likely in frontotemporal dementia than in Alzheimers disease. None of the features of dementia with Lewy
bodies (hallucinations, parkinsonism, fluctuating symptoms) is present. Creutzfeldt-Jakob disease evolves
more rapidly, and usually causes myoclonus and other neurologic signs.
Neurology:Question 33
The correct answer is E
Educational Objectives
Recognize the effect of corticosteroids on primary lymphoma of the CNS.
Critique
This is an elderly patient with subacute progressive neurologic deficits referable to the supratentorial level and
signs of raised intracranial pressure. The clinical history and imaging suggest neoplasm. Corticosteroid therapy
produces rapid clinical and radiographic improvement. This clinical picture is most characteristic of primary
central nervous system lymphoma. If primary central nervous system lymphoma is suspected, it is important to
refrain from using corticosteroids prior to biopsy, unless the patients clinical situation absolutely requires it.
Corticosteroids exert an oncolytic effect on lymphomas and may render the biopsy nondiagnostic.
Glioblastoma and metastatic carcinoma are unlikely to have such a dramatic radiographic and clinical response
to corticosteroids. Although vasculitis or multiple sclerosis may respond rapidly to corticosteroids, neither
disorder is likely to develop in a 70-year-old patient, and neither would generally present as a mass lesion on
brain imaging.
Neurology:Question 34
The correct answer is E
Educational Objectives
Treat secondary progressive multiple sclerosis (MS).
Critique
Mitoxantrone is approved by the U.S. Food and Drug Administration (FDA) to slow progression of secondary
progressive or severe relapsing-remitting MS. The speed of this patients decline merits consideration of
aggressive immunosuppression. The primary concern about treatment with mitoxantrone is that dose-related
cardiotoxicity prohibits use beyond a cumulative dose of 120 to 140 mg/m 2.
No benefit has been shown for chronic prednisone treatment and this is not recommended. Bone marrow
transplantation is investigational only. There is some evidence that plasmapheresis is useful for acute
devastating attacks of demyelination that do not respond quickly to standard intravenous corticosteroid
treatment, but there is little evidence of benefit for progressive disease. Glatiramer acetate can reduce the
frequency of relapses in patients with relapsing-remitting MS, but is not indicated for progressive disease.
Results are inconsistent concerning whether interferon beta slows secondary progressive MS. Other agents that
have been used for secondary progressive disease include cyclophosphamide, intravenous immunoglobulin,
azathioprine, and methotrexate, although none are FDA approved for this purpose.
Neurology:Question 35
The correct answer is A
Educational Objectives
Select prophylaxis for migraine headache.

Critique
This patient has frequent attacks of migraine without aura. She is at risk for developing rebound headaches
from overuse of acute medication. Accordingly, a preventive medication is indicated. Amitriptyline is a first-line
agent for migraine prophylaxis; its sedating effect may also help her insomnia. Nadolol and propranolol are
contraindicated because they may trigger her asthma. Naratriptan is used for treatment of acute migraine and
is not approved for prophylactic therapy. Selective serotonin reuptake inhibitors have limited effectiveness in
migraine prophylaxis.
Neurology:Question 36
The correct answer is B
Educational Objectives
Determine when to withdraw antiepileptic drug treatment.
Critique
According to recent studies of adult patients who are seizure free for more than 2 years, approximately one
third relapse following termination of antiepileptic drugs. Risk factors associated with recurrence of seizures
include prolonged duration and high frequency of seizures before they were controlled, abnormal neurologic
examination, abnormal magnetic resonance imaging (MRI) of the brain, consistently abnormal
electroencephalograms (EEG5), and mental retardation. This patient has consistently abnormal EEGs. He
should continue antiepilepsy treatment despite having been seizure free for 4 years. Ordering another EEG and
MRI would not change the outcome. Single photon emission computed tomography (SPECT) is not helpful
when clinical history, EEG, or MRI provide sufficient localization. Changing to another agent such as
carbamazepine would be appropriate if the patient had side effects from phenytoin.
Neurology:Question 37
The correct answer is C
Educational Objectives
Recognize the typical clinical presentation of cerebral meningioma.
Critique
This 50-year-old patient has a chronic progressive history of a scalp/skull mass and symptoms of raised
intracranial pressure. Computed tomography demonstrates an extra-axial calcified enhancing mass with
overlying skull hyperostosis, a classic picture for meningioma. Meningiomas derive from the arachnoid
granulations of the dura, hence their usual occurrence over the cerebral convexities, skull base, or falx cerebri.
Calcification is typical and reflects the slow-growing, chronic nature of the tumor. Meningiomas often invade
adjacent bone and may produce a palpable or visible hyperostotic mass. Meningiomas most commonly affect
middle-aged and elderly women.
Gliomas, including glioblastoma multiforme, are intra-axial tumors that involve the parenchyma of the brain
and not the dural surface. Dural metastases are by definition malignant tumors and usually present more
acutely. They do not calcify. While chronic subdural hematomas may calcify, they usually do not enhance, nor
do they produce hyperostosis of the overlying bone.
Neurology:Question 38
The correct answer is B
Educational Objectives
Treat an elderly patient with a large cerebral infarction.
Critique
Intravenous tissue plasminogen activator (t-PA) decreases the risk of disability in patients with ischemic stroke
when administered within 3 hours of stroke onset (National Institute of Neurological Disorders and Stroke
[NINDS] protocol). Although older patients have a higher risk of intracerebral hemorrhage than younger
patients, persons within the same age group who receive intravenous t-PA fare better than those who do not.
The same is true for patients with large cerebral infarctions (for example, infarcts involving more than one third
of the middle cerebral artery territory). At this time, there is insufficient evidence to recommend intra-arterial
over intravenous t-PA within 3 hours of stroke onset, and intra-arterial t-PA is not currently approved by the
U.S. Food and Drug Administration for ischemic stroke. The NINDS protocol forbids administration of aspirin
or any other antithrombotic agent within 24 hours after t-PA administration. To date, all clinical trials
assessing emergent administration of aspirin plus thrombolysis in stroke patients have shown poor outcomes.
Neurology:Question 39
The correct answer is A
Educational Objectives
Manage status epilepticus.
Critique
The patient has already received a full dose of lorazepam (0.1 mg/kg). The most appropriate next step is to
administer a full loading dose of fosphenytoin or phenytoin (20 mg/kg of body weight; the phenytoin
equivalence in option C is too low). Fosphenytoin is preferred over phenytoin because it can be administered

faster than phenytoin and has a lower incidence of hypotension and local thrombophlebitis at the site of
administration (purple glove syndrome). Intravenous infusion of fosphenytoin (ordered in phenytoinequivalent doses) should be initiated while lorazepam is still being administered because it takes approximately
15 minutes for fosphenytoin to take effect. If there is no risk of respiratory suppression (patient is intubated)
and the patient is hemodynamically stable, additional doses of lorazepam (beyond a full dose) can be given if
necessary. Appropriate loading doses (mg/kg of body weight) of phenobarbital (after intubation) or valproic
acid can be administered if the patient continues to have seizures despite treatment with lorazepam and
fosphenytoin. The common practice of routinely giving 1 gram of phenytoin may result in under- or overdosing.
Phenytoin should be administered according to patient weight.
Neurology:Question 40
The correct answer is D
Educational Objectives
Recognize dementia with Lewy bodies.
Critique
The patient manifests the cardinal features of dementia with Lewy bodies, including cognitive impairment,
fluctuating symptoms, hallucinations, and parkinsonism. Patients who have dementia with Lewy bodies often
also have a cholinergic deficit, as suggested in this case by the episode of confusion associated with an
anticholinergic medication. The fluctuating and psychiatric symptoms warrant treatment. Treatment with a
cholinesterase inhibitor, such as donepezil, may be beneficial.
Mild cognitive impairment is not an appropriate diagnosis, because the disorder is severe enough to impair his
daily function, and because the deficits go beyond memory impairment. A primary diagnosis of Alzheimers
disease is not appropriate because fluctuating sensorium and parkinsonism are not core features of this disease,
as they are in dementia with Lewy bodies. However, a component of Alzheimer pathology is commonly
superimposed on the pathology of dementia with Lewy bodies. For similar reasons, a primary diagnosis of
frontotemporal dementia is also not appropriate.
Neurology:Question 41
The correct answer is A
Educational Objectives
Distinguish chorea from other hyperkinetic movement disorders.
Critique
Rapidly flowing, dancing-like involuntary movements occurring in a chaotic, nonstereotyped fashion define
chorea. The differential diagnosis for chorea includes disorders that are inherited (for example, Huntingtons
disease), immune-mediated (for example, antiphospholipid antibody or paraneoplastic syndrome),
endocrinologic (for example, hyperthyroidism), and hematologic (for example, polycythemia vera or
neuroacanthocytosis). In this case, the patients age, chronicity of symptoms, and abnormal mental status score
suggest Huntingtons disease. This could be confirmed by genetic testing, if he agrees.
Chorea does not occur in Parkinsons disease unless provoked by medications (he is taking none). The 3-year
course makes Creutzfeldt-Jakob disease unlikely. Tourettes syndrome starts earlier in life with stereotyped
movements (tics). This patients adventitious movements are chaotic and not stereotyped like tics.
Neurology:Question 42
The correct answer is D
Educational Objectives
Classify course of multiple sclerosis.
Critique
This womans course began with acute exacerbations followed by remissions of variable duration, so her disease
was initially relapsing-remitting. However, in the past year, she has had progression of neurologic deficits
between acute relapses. In at least 50% of patients with relapsing-remitting multiple sclerosis, disease will
evolve to a secondary progressive course, such as in this case. Even a single remote attack followed a decade
later by the onset of progression is classified as a secondary progressive course.
Monosymptomatic demyelination refers to a single clinical episode of demyelination, such as isolated optic
neuritis. Relapsing-remitting disease refers to multiple attacks of demyelination. Primary progressive multiple
sclerosis starts as an insidious spinal cord dysfunction or as ataxia, progressing without distinct acute
exacerbations; it tends to have fewer cerebral lesions (as seen on magnetic resonance imaging) than other
forms of multiple sclerosis. Benign multiple sclerosis refers to a course in which all neurologic systems are fully
functional 15 years after disease onset.
Neurology:Question 43
The correct answer is D
Educational Objectives
Manage symptomatic extracranial carotid atherosclerosis.
Critique

Sudden right hemiparesis with expressive aphasia suggests a left cortical infarction in the territory of the left
middle cerebral artery, which is supplied by the left internal carotid artery. Carotid stenosis of 50% or more that
is ipsilateral to the infarction implies that the stroke is due to large artery disease. Carotid endarterectomy
decreases risk of future stroke in patients with symptomatic stenosis of 50% or more, and especially in patients
with 70% to 99% stenosis, as compared with antiplatelet agents alone. Aspirin, 81 or 325 mg daily, decreases
the risk of stroke after endarterectomy. There is no evidence that intravenous heparin is safe and effective in
patients with acute ischemic stroke.
Neurology:Question 44
The correct answer is D
Educational Objectives
Manage small, asymptomatic meningiomas.
Critique
This 75-year-old woman has what is most probably a small, asymptomatic convexity meningioma. The
calcification suggests that the meningioma has been present for months or years. Calcified meningiomas are
less likely to progress than noncalcified lesions. In this setting, the appropriate management is follow-up
imaging in 6 months to assess the stability of the lesion, with consideration of further follow-up at increasing
intervals if no growth is seen. If the lesion enlarges or becomes symptomatic, then surgical resection is
indicated, assuming the patient is a suitable candidate. Biopsy and radiosurgery are not appropriate for patients
with incidental, asymptomatic meningioma. In autopsy series, small, asymptomatic meningiomas have been
found in 1% to 2% of individuals.
Neurology:Question 45
The correct answer is D
Educational Objectives
Recognize the persistent vegetative state.
Critique
In this patient, severe cerebral anoxia from her cardiac arrest led to diffuse cortical injury with relative
preservation of brainstem function, and the development of a vegetative state. A vegetative state is a clinical
condition of complete unawareness of self or the environment, accompanied by sleep-wake cycles and
preservation of brainstem and hypothalamic autonomic functions. According to the conclusions of a multisociety task force, persistent vegetative state is diagnosed if a vegetative state is present 1 month after acute
traumatic or nontraumatic brain injury. Unlike a vegetative state, in which there is no evidence of
consciousness, in the minimally conscious state there is severely altered consciousness with minimal but
definite behavioral evidence of self or environmental awareness. In the locked-in syndrome, patients are
conscious because of normal cortical and upper brainstem function, but are quadriplegic and can communicate
only through the ability to move their eyes vertically and blink; this syndrome is caused by lesions at the base of
the pons. Brain death is the complete absence of all cerebral hemispheric and brainstem functions, including
the absence of respiratory drive.
Neurology:Question 46
The correct answer is A
Educational Objectives
Treat optic neuritis.
Critique
This patient has signs and symptoms consistent with left optic neuritis. Although most patients have good
recovery in 6 months with or without treatment, a course of intravenous methylprednisolone (1 g daily for 3
days) followed by an oral prednisone taper speeds recovery of visual acuity in optic neuritis. The decision on
whether to use corticosteroids or hold treatment depends on patient preference, severity of the visual deficit,
health of the fellow eye, and vocational requirements.
This patient has a single clinical demyelinating event, with no history of previous clinical neurologic events or
evidence of silent demyelinating lesions on brain imaging. Therefore, a diagnosis of multiple sclerosis and
treatment with disease-modifying therapies (interferon beta or glatiramer acetate) are not warranted. However,
surveillance may be appropriate with follow-up magnetic resonance imaging of the brain to watch for
development of cerebral white matter lesions. One major study of optic neuritis found that there is no benefit to
treatment with oral prednisone alone over no treatment; indeed, the study found an increased risk of optic
neuritis in the fellow eye in the next 6 to 12 months with oral prednisone treatment. The presentation is not
suggestive of an ischemic event and there is no indication for treatment with aspirin.
Neurology:Question 47
The correct answer is D
Educational Objectives
Manage complex partial seizures.
Critique

This patients episodes are most likely complex partial seizures of frontal lobe origin. Frontal lobe
seizures are brief, usually occur in clusters, and usually are not associated with aura or postictal confusion.
Appropriate anticonvulsant management should be initiated, even during the diagnostic work-up.
Carbamazepine is approved by the U.S. Food and Drug Administration (FDA) for treatment of partial seizures
and is an appropriate initial choice. Phenobarbital is not a first-line agent because of possible associated
sedation and cognitive impairment.
Gabapentin is approved for adjunctive treatment of partial seizures but should not be used in monotherapy.
Ethosuximide is used to treat absence seizures, which this patient does not have. Lamotrigine is FDA approved
for adjunctive treatment of partial seizures; many neurologists use it with success as second-line monotherapy.
Neurology:Question 48
The correct answer is C
Educational Objectives
Treat essential tremor.
Critique
This patient has a postural and action tremor (essential tremor), rather than a resting tremor typical of
Parkinsons disease. First-line drugs include primidone or a nonselective -adrenergic blocker, such as
propranolol. Selective 1 blockers, such as atenolol, are not effective against tremor, at least in the lower doses
where the 1 effect is predominant. As the clinical description is not that of parkinsonian tremor,
carbidopa/levodopa or pramipexole would not be helpful. Vaiproic acid may induce tremor and is not used to
treat it.
Neurology:Question 49
The correct answer is A
Educational Objectives
Prescribe an angiotensin-converting enzyme (ACE) inhibitor and a diuretic for patients who have had stroke or
transient ischemic attack, regardless of baseline blood pressure.
Critique
In PROGRESS, patients with a recent stroke or transient ischemic attack (TIA) had decreased risk of future
stroke, heart attack, or death if they took the ACE inhibitor perindopril and the diuretic indapamideeven if
they had no previous history of hypertension or if their pretreatment blood pressure levels were in the so-called
normotensive range. However, it is best not to lower blood pressure aggressively soon after a stroke in order
to preserve the penumbra and because cerebral autoregulation is dysfunctional. In PROGRESS, low-dose
perindopril was begun 2 weeks after the stroke, and gradual increases in antihypertensive therapy were made at
2-week intervals. The results of recent clinical outcome-based trials such as PROGRESS have made it
increasingly clear that optimal target blood pressure levels are lower than we previously believed. The target
blood pressure for patients with diabetes or end-organ damage, including stroke patients, is 130/80 mm Hg.
There are no data regarding the efficacy of -blockers or calcium channel blockers for secondary stroke
prevention.
Neurology:Question 50
The correct answer is E
Educational Objectives
Select the appropriate medication for acute treatment of migraine.
Critique
This patient has episodic severe migraine without aura. Zolmitriptan and other triptans are more likely to be
effective for severe migraine headaches than nonspecific agents such as combination analgesics and
nonsteroidal anti-inflammatory drugs. Nonspecific agents may be effective for mild to moderate headaches.
Neurology:Question 51
The correct answer is B
Educational Objectives
Recognize the association of Epstein-Barr infection with primary lymphoma of the central nervous system in
immunocompromised patients.
Critique
Primary central nervous system (CNS) lymphoma develops in approximately 2% of patients with HIV infection,
usually late in the course of infection. Unlike the lesions of immunocompetent patients with primary CNS
lymphoma, the lesions in HIV-infected patients often manifest central necrosis with a ring-enhancing
appearance that is difficult to distinguish from CNS infections such as toxoplasmosis. Nearly all primary CNS
lymphomas in HIV-infected individuals contain Epstein-Barr viral genome. Cerebrospinal fluid polymerase
chain reaction is typically negative for Epstein-Barr virus. In the presence of reduced T-cell function, the
Epstein-Barr-infected B cells become immortalized and are driven toward a monoclonal malignant B-cell
population. Epstein-Barr viral genome is rare in primary CNS lymphoma in immunocompetent patients.
Patients with primary CNS lymphoma may be treated with high-dose methotrexate and brain radiation.

Unfortunately, treatment of primary CNS lymphoma in patients with HIV is rarely successful, and most
patients die within 6 months of diagnosis (usually from complications of other opportunistic infections).
Cytomegalovirus, herpes zoster, and herpes simplex may infect the brain of an immunocompromised patient,
but none is associated with CNS lymphoma.
Neurology:Question 52
The correct answer is A
Educational Objectives
Treat a patient with subarachnoid hemorrhage.
Critique
This patient has a subarachnoid hemorrhage, likely due to aneurysm rupture. In terms of the hemorrhage, the
most important intervention at this time is to initiate treatment with nimodipine. Nimodipine helps reduce
subsequent development of vasospasm and has been shown to improve outcome in clinical trials. Other
antihypertensive medications, such as labetalol and enalapril, are not necessary because nimodipine has an
antihypertensive effect. There is no role for corticosteroids in patients with subarachnoid hemorrhage.
Neurology:Question 53
The correct answer is B
Educational Objectives
Diagnose mononeuropathy multiplex.
Critique
This patient has a right peroneal mononeuropathy and a left ulnar mononeuropathy. By definition, this is a case
of multiple mononeuropathies (mononeuritis multiplex). Mononeuritis multiplex is usually due to vascular
infarction of multiple nerves. Causes of mononeuritis multiplex include diabetes (unlikely in this case because
of the normal fasting glucose), connective tissue diseases (such as systemic lupus erythematosus or rheumatoid
arthritis), polyarteritis nodosa, Wegeners granulomatosis, HIV infection, and isolated peripheral nervous
system vasculitis. The systemic findings in this case suggest connective tissue disease or systemic necrotizing
vasculitis.
The effects of amyloidosis on the peripheral nervous system typically involve the autonomic nerves and may be
associated with bilateral carpal tunnel syndrome. Guillain-Barr syndrome causes acute symmetric limb or
bulbar weakness often preceded by sensory symptoms, but would not cause multiple asymmetric
mononeuropathies, as is the case in this patient.
Neurology:Question 54
The correct answer is D
Educational Objectives
Recognize juvenile myoclonic epilepsy.
Critique
The patient had myoclonic jerks that led to a generalized tonic-clonic seizure. This is a typical presentation of
juvenile myoclonic epilepsy, an idiopathic epilepsy syndrome. Affected patients have a normal neurologic
examination and normal development. Unlike other idiopathic epilepsies, remission is rare even after years
without a seizure, so lifelong treatment is necessary. Patients are sensitive to sleep deprivation, alcohol, and
stress.
Unlike juvenile myoclonic epilepsy, which is a primary, generalized epilepsy, partial seizures originate from a
focal part of the brain, and their clinical presentation depends on their neuroanatomic location. Absence
seizures mainly affect children. They manifest with sudden onset of staring, with or without eye blinking or lip
smacking. Patients with syncope often have several myoclonic jerks while unconscious, but they recover quickly
once blood pressure is reestablished. Nonepileptic events (pseudoseizures) vary in presentation, but they are
often associated with moaning, crying, and arrhythmic, on-and-off shaking of the body.
Neurology:Question 55
The correct answer is B
Educational Objectives
Assess effect of immunomodulatory therapy on pregnancy.
Critique
Interferon beta is teratogenic in animals; spontaneous abortion is a risk. This patient should be advised to stop
interferon beta treatment immediately and see her obstetrician for evaluation and monitoring of fetal
development. Women taking disease-modifying treatment for multiple sclerosis should be advised to practice
effective contraception and to stop interferon or glatiramer acetate several months before attempting to
conceive. There have, however, been successful pregnancies with early exposure to interferon beta.
Pregnancy has a short-term effect on the course of multiple sclerosis. Typically, the frequency of relapse
decreases significantly in the third trimester, with a rebound increase in the first 6 months post partum. No
studies have demonstrated a long-term effect of pregnancy on disability progression. First-degree relatives of a
person with multiple sclerosis have a risk of disease 20 to 50 times greater than that of the general population.

However, the transmission of increased susceptibility is believed to be multigenic; no simple one-gene pattern,
such as autosomal-recessive transmission, is observed. There is no medical indication for therapeutic abortion
in this patient.
Neurology:Question 56
The correct answer is D
Educational Objectives
Diagnose hemicrania continua.
Critique
Hemicrania continua is a primary headache disorder characterized by chronic, daily unilateral headache with
some autonomic phenomena. Indomethacin is effective in managing the disorder; other preventive agents are
usually ineffective. Cluster headaches last 1 to 2 hours and predominantly occur in men. Chronic tension-type
headaches are usually bilateral and do not have associated autonomic phenomena. Chronic migraine is typically
preceded by years of episodic migraine.
Neurology:Question 57
The correct answer is B
Educational Objectives
Assess iron levels in patients with restless legs syndrome.
Critique
This patients description is typical of restless legs syndrome. Because restless legs syndrome may occasionally
be a manifestation of iron deficiency, it is appropriate to check iron levels. If deficiency is identified, iron
supplementation may resolve symptoms. Otherwise, restless legs syndrome usually responds to a dopamine
agonist medication (such as pergolide, pramipexole, or ropinirole) or gabapentin. Carbidopa/levodopa is
effective against restless legs syndrome, but it is second-line therapy because of an associated rebound
phenomenon (augmentation).
Typically, obtaining the serum ferritin level is an adequate work-up if the clinical picture fits with restless legs
syndrome, the neurologic examination is normal, and there are no major medical problems. Hence, magnetic
resonance imaging of the brain is unnecessary. Serum ceruloplasmin or urine porphyrin levels are not relevant
to this clinical presentation. Nerve conduction studies are not helpful in the diagnosis of restless legs syndrome,
and this patient has no findings on examination to suggest neuropathy.
Neurology:Question 58
The correct answer is A
Educational Objectives
Recognize Creutzfeldt-Jakob disease and identify the appropriate confirmatory test.
Critique
This patient has signs of a subacute dementing illness. The pace of the illness, normal imaging study, and
presence of myoclonus suggest Creutzfeldt-Jakob disease. Treatable inflammatory conditions, such as
granulomatous angiitis of the nervous system, are unlikely but remain in the differential diagnosis, and brain
biopsy might be required to exclude them. Before considering brain biopsy, a positive diagnosis of CreutzfeldtJakob disease can be made if there are periodic sharp waves on the electroencephalogram or if 14-3-3 protein is
present in the cerebrospinal fluid. Although the electroencephalogram in this case might evolve over time and
become diagnostic, the pace of disease progression makes waiting inadvisable.
Functional imaging can localize dysfunctional tissue, but not characterize it. The presenilin test is indicated
only when familial Alzheimer-profile dementia is suspected.
Neurology:Question 59
The correct answer is C
Educational Objectives
Recognize the association of specific tumors with paraneoplastic neurologic disease.
Critique
This patient has paraneoplastic cerebellar degeneration. Paraneoplastic neurologic disorders are rare
complications of cancer. In this patient, a structural lesion of the posterior fossa, such as a tumor or infarct, has
been appropriately excluded by imaging.
In each of the paraneoplastic syndromes, the systemic tumor cell is believed to express an onconeural antigen
that produces an immune response in the patient. This onconeural antigen shares similarities to antigens
normally expressed by specific neural tissue. The host immune response (cell mediated and humoral) then
attacks both the tumor and the specific neural tissue that share antigenic similarity. This theory is strengthened
by the fact that patients tumors often share antigenic similarities with neural tissue and by the frequent
observation that patients with paraneoplastic neurologic disease often have limited or no metastatic disease
and small primary tumors. The latter observation suggests that the immune response to tumor may be
particularly strong in these patients.
The presence in this patients serum of the anti-Yo antibodies is very specific for paraneoplastic cerebellar

degeneration in the setting of ovarian, uterine, fallopian tube, or breast cancer. Anti-Hu (anti-neuronal
antibody type I) is the antibody associated with small cell lung cancer and paraneoplastic neurologic disease.
Paraneoplastic neurologic disorders are very rare in non-Hodgkins lymphoma or melanoma, and there is no
serologic antibody specifically associated with either tumor.
Neurology:Question 60
The correct answer is C
Educational Objectives
Recognize frontal lobe seizures.
Critique
The patient most likely has simple partial seizures of frontal lobe origin. The clinical presentation of partial
seizures depends on their neuroanatomic location. In this case, the seizures originate in Brocas language area
and spread to the primary motor area. Frontal seizures are brief and usually without any aura or postictal
confusion. The recent onset of seizures should lead to brain imaging as soon as possible to rule out a spaceoccupying lesion. Transient ischemic attack, hemiplegic migraine, and hypoglycemia can cause focal neurologic
deficits, but would not cause twitching of the face and arm. Anxiety attacks last longer (minutes) and are
associated with other symptoms, such as shortness of breath and fear.
Neurology:Question 61
The correct answer is B
Educational Objectives
Diagnose aneurysmal subarachnoid hemorrhage.
Critique
This young patient has sudden onset of severe, debilitating headache without prior history of such events. It is a
classic first, worst headache scenario that suggests subarachnoid hemorrhage. Further work-up is needed to
evaluate for subarachnoid blood. Computed tomography (CT) is sensitive for detecting acute hemorrhage, but
its ability to detect subarachnoid hemorrhage is not absolute, and after 3 days, 25% of affected patients have
normal CT scans. When subarachnoid hemorrhage is clinically suspected despite a normal CT scan, lumbar
puncture is imperative to rule it out. In general, magnetic resonance imaging is inferior to CT for detecting
subarachnoid blood. Arteriography is not warranted acutely until the presence of subarachnoid hemorrhage has
been established. Abortive treatment for headache should be given only after subarachnoid hemorrhage has
been ruled out.
Neurology:Question 62
The correct answer is D
Educational Objectives
Treat acute migraine.
Critique
This patient has migraine. An oral medication will have limited efficacy, because her headaches are
accompanied by early nausea and vomiting. Sumatriptan is the only 5HT1 B,D agonist available in a parenteral
form and is highly effective. Nasal sumatriptan is another reasonable choice, but it is generally less effective.
Subcutaneous or intramuscular dihydroergotamine could also be considered for acute treatment, but
intravenous administration is not appropriate in the home setting.
Neurology:Question 63
The correct answer is B
Educational Objectives
Diagnose mild cognitive impairment.
Critique
This patients abnormal finding is limited to memory, and he has no impairment of interpersonal, occupational,
or daily living activities. Therefore, his condition should be classified as mild cognitive impairment, not
dementia. Follow-up evaluation in 1 year is appropriate, as many affected patients may have pre-demential
Alzheimers disease. The conversion rate from mild cognitive impairment to mild dementia is 10% to 15% per
year.
Although there may be an autosomal-dominant form of Alzheimers disease in this patients family, affected
members had disease onset when they were 20 years younger than this patient. Thus, if the patient has
Alzheimers disease, it is more likely to be a sporadic than familial case, and genetic testing for presenilin-1 is
not likely to be helpful. The specificity of cerebrospinal fluid findings and functional imaging studies (such as
positron emission tomography/single photon emission computed tomography [PET/SPECT]) for early
dementia has not been determined, so it is unlikely that these tests would be helpful. Diffusion-weighted
magnetic resonance imaging is sensitive for acute ischemic stroke and has no role in evaluating degenerative
disease.

Neurology:Question 65
The correct answer is B
Educational Objectives
Use botulinum toxin to treat adult-onset focal dystonia.
Critique
The patient has cervical dystonia (spasmodic torticollis), an idiopathic focal dystonia of adulthood. Although its
origins are presumed to be within the brain, imaging studies of the brain are negative, as are studies of the
cervical spine. Oral medications for cervical dystonia, including muscle relaxants (diazepam, cyclobenzaprine),
are mildly effective at best. Analgesics may be necessary to relieve pain from muscle contraction. The most
consistently effective medical treatment is botulinum toxin injected into appropriate neck muscles, sometimes
under electromyographic guidance. Within a few days, the injected muscles weaken, tending to release the
spasm. Benefits persist for 2 to 4 months, and reinjection every 3 or more months maintains the benefits.
Excessive weakness of the injected muscles is rarely a problem.
Although carbidopa/levodopa may improve the dystonia of Parkinsons disease, it is ineffective in treating
primary idiopathic focal dystonias, such as cervical dystonia. Psychiatry referral may improve stress
management, but is unlikely to have a major impact on the motor manifestations of cervical dystonia.
Neurology:Question 66
The correct answer is A
Educational Objectives
Recognize complex partial seizures of temporal lobe origin.
Critique
This is a typical description of complex partial seizure of temporal lobe origin. The symptoms of complex partial
seizures vary, but they are stereotypical for each patient. Patients may have a warning (aura) prior to the
seizure. The most common warning is a rising epigastric sensation; other auras may include affective (such as
fear), cognitive (such as dj vu), and sensory symptoms (such as olfactory hallucinations). Complex partial
seizures typically last less than 3 minutes. During that time the patient appears awake, but loses contact with
the environment and does not respond normally to instructions or questions. Patients usually stare and remain
motionless, or engage in repetitive, semi-purposeful behavior (automatisms), such as gesturing, chewing, lip
smacking, repeating words or phrases, walking, running, or undressing. Patients do not recall these behaviors.
After the seizure the patient is confused or sleepy for minutes to hours.
Absence seizures occur primarily in children, are not preceded by aura, and are not followed by postictal
confusion. They last 5 to 10 seconds and occur primarily in clusters. Nonepileptic events (pseudoseizures) are
characteristically variable (not stereotypical) in presentation and last longer. Migraine can be associated with
neurologic symptoms, including confusion, but there is usually a history of headache or other obvious symptom
(for example, visual phenomena). Transient global amnesia is a syndrome of acute memory loss of unknown
etiology; it can last for minutes to hours. There is no alteration or loss of consciousness during transient global
amnesia.
Neurology:Question 67
The correct answer is B
Educational Objectives
Recognize the clinical presentation of Lambert-Eaton myasthenic syndrome.
Critique
This patient has Lambert-Eaton myasthenic syndrome (LEMS), which in 60% of cases occurs in patients
harboring small cell lung cancer. LEMS is a disorder of nicotinic and muscarinic cholinergic synapses resulting
in impaired acetylcholine release, with muscle weakness and autonomic dysfunction. Affected patients produce
antibody to the voltage-gated calcium channel (VGCC), thus impairing calcium influx into the nerve terminal
and reducing acetylcholine release. The circulating anti-VGCC antibody is detectable in some but not all
patients. Spinal cord compression presents with spine and radicular pain and evidence of myelopathy (signs of
upper motor neuron weakness, spasticity, Babinski signs, and sensory loss typically with a defined level).
Polymyositis generally produces muscle pain and weakness without symptoms of autonomic nervous system
dysfunction. Polyneuropathy is a possibility, although it usually would produce distal more than proximal
weakness; the weakness would not improve with repeated effort. Autonomic dysfunction is less commonly
associated with polyneu ropathy.
Neurology:Question 68
The correct answer is C
Educational Objectives
Treat acute migraine in the emergency department.
Critique
This patient needs parenteral treatment for her prolonged, severe migraine headache that is accompanied by
repeated vomiting. The neuroleptic agent chlorpromazine is effective for such a situation. Although

dihydroergotamine may be useful when administered intravenously during a severe migraine attack, it should
not be used in this situation because the patient has already used rizatriptan within the past 24 hours and both
medications have vasoconstrictive effects. Dihydroergotamine may cause nausea and vomiting, so when it can
be used intravenously, an antiemetic such as metoclopramide must be administered first. Oral codeine (or any
oral agent) is a poor choice since the patient is nauseated and vomiting. Finally, sumatriptan should not be used
because the patient has already used rizatriptan within the past 24 hours.
Neurology:Question 69
The correct answer is A
Educational Objectives
Prescribe cholesterol-lowering therapy for secondary prevention of stroke.
Critique
Among patients with known coronary artery disease, statin drugs decrease the risk of ischemic stroke as well as
myocardial infarction. The evidence is strongest for pravastatin, 40 mg/d, and simvastatin, 20 to 40 mg/d.
Previous studies of lipid-lowering therapy other than statins did not show a decrease in stroke risk (and showed
lesser decreases in cholesterol levels). Observational studies suggest that a low-density lipoprotein (LDL) level
below 70 mg/dL may increase the risk of hemorrhagic stroke. Patients with diabetes or a history of stroke,
coronary artery disease, or peripheral artery disease should take a statin, if at all possible, to achieve a target
LDL value between 70 and 100 mg/dL. LDL cholesterol levels decrease in the first 24 to 48 hours after an
ischemic event and do not return to baseline for several weeks. It is likely that this patients LDL value of 110
mg/dL in the hospital is lower than his pre-event value, further emphasizing his need for lipid-lowering
therapy.
Neurology:Question 70
The correct answer is B
Educational Objectives
Recognize treatment options for medically intractable epilepsy.
Critique
This patient has a seizure disorder of right temporal lobe origin. Epilepsy is considered medically intractable
when adequate trials of three different antiepileptic drug regimens have not controlled seizures. Excision of the
epileptogenic focus is now widely accepted for medically intractable seizure disorders, particularly those of
temporal lobe origin.
Vagal nerve stimulation is less efficacious than surgery for medically intractable epilepsy, but should be
considered if the patient is not a candidate for temporal lobectomy. Adding a third or fourth antiepileptic drug
would probably cause more side effects without significantly improving seizure control. Positron emission
tomography is not necessary, because there is enough clinical, electroencephalographic, and radiographic
evidence that the patients seizures originate from the right temporal lobe. Repeat magnetic resonance imaging
of the brain is not necessary, since mesial temporal sclerosis does not usually progress, and there is no
correlation between the degree of sclerosis and seizure frequency.
Neurology:Question 72
The correct answer is D
Educational Objectives
Use immunomodulatory therapy in multiple sclerosis.
Critique
This patient has clinically definite relapsing-remitting multiple sclerosis. The current consensus of the National
Multiple Sclerosis Society is that all patients with definite relapsing multiple sclerosis be considered for
treatment with either a form of interferon beta or glatiramer acetate. These immunomodulatory agents
decrease the frequency of relapse and the rate of accumulation of new lesions on magnetic resonance imaging.
High-dose intravenous methylprednisolone (for example, 500 to 1000 mg daily for 3 to 5 days) is standard
treatment for a significant acute exacerbation of multiple sclerosis. Periodic pulses of high-dose corticosteroids
are also given to some patients with progressive disease. However, this treatment is not indicated for patients
who are stable between attacks. Chronic oral prednisone is not used in multiple sclerosis, as there is no
indication that its benefits outweigh its toxicity. Azathioprine was used to treat relapsing multiple sclerosis in
the past, but since the development of interferon, it is used only as an investigational therapy for progressive
multiple sclerosis.
Neurology:Question 73
The correct answer is D
Educational Objectives
Diagnose idiopathic intracranial hypertension.
Critique
Idiopathic intracranial hypertension most commonly occurs in obese young women and is usually manifested

by an unremitting headache; there may be associated visual obscurations. The neurologic examination is
typically normal except for papilledema and enlarged blind spots. Its most serious long-term complication is
vision loss. The elevated cerebrospinal fluid opening pressure could be due to hydrocephalus, but this diagnosis
is excluded by the normal findings on magnetic resonance imaging of the brain. There is no spinal fluid
pleocytosis, so meningitis is unlikely in this patient. Chronic migraine is not associated with elevated
cerebrospinal fluid pressure and papilledema. The patient is not using medication at a frequency that would
lead to medication overuse (analgesic rebound) headache, nor is medication overuse headache associated with
evidence of increased intracranial pressure.
Neurology:Question 74
The correct answer is B
Educational Objectives
Recognize status epilepticus.
Critique
This patients clinical presentation is consistent with partial status epilepticus. The patient has been having
complex partial seizures of right frontal lobe origin without recovery of baseline mental status between seizures.
The seizure in the hospital was probably a partial seizure with secondary generalization due to her stroke. She
requires urgent neurologic consultation and urgent electroencephalography to confirm partial status
epilepticus. The patients neurologic examination is consistent with the previous history of right hemispheric
stroke. The worsening of the left-sided weakness is more likely due to the frequent seizures (Todds paralysis)
than to stroke in evolution. Phenytoin toxicity can worsen a previous neurologic deficit, but is unlikely to cause
more seizures. Creutzfeldt-Jakob disease is a subacute, degenerative disorder characterized by rapidly
progressive dementia, ataxia, and myoclonic jerks; it is unlikely in this patient.
Neurology:Question 75
The correct answer is A
Educational Objectives
Recognize medication-induced cognitive dysfunction.
Critique
The patients altered level of consciousness and asterixis suggest a toxic/metabolic etiology of the impairment.
The patient is on several medications that might impair cognition. Nortriptyline has anticholinergic properties,
and is especially likely to cause impairment in patients with latent cholinergic deficiency (for example, patients
with early dementia or Parkinsons disease). The sedative-hypnotic lorazepam and digoxin may also contribute
to cognitive impairment.
Dementia with Lewy bodies is also associated with fluctuation of cognitive status and visual hallucinations, but
is less likely given the medication history, absence of parkinsonism, and presence of asterixis. Vascular
dementia, particularly that due to small-vessel cerebrovascular disease, is unlikely given the absence of a
stepwise course and absence of stroke on computed tomography. Depression may cause chronic cognitive
impairment (pseudodementia), but not asterixis and an altered level of consciousness.

También podría gustarte